2016 AMC 10/12 B Discussion
Go back to the Math Jam ArchiveA discussion of problems from the AMC 10/12 B, which was administered February 17. We will cover the last 5 problems on each test, as well as requested earlier problems on the test. (Note: this date has been updated from the previous date of Feb 22.)
Copyright © 2024 AoPS Incorporated. This page is copyrighted material. You can view and print this page for your own use, but you cannot share the contents of this file with others.
Facilitator: Jeremy Copeland
copeland
2016-02-26 19:03:53
Welcome to the 2016 AMC 10B/12B Math Jam!
Welcome to the 2016 AMC 10B/12B Math Jam!
copeland
2016-02-26 19:03:54
I'm Jeremy Copeland, and I'll be leading our discussion tonight.
I'm Jeremy Copeland, and I'll be leading our discussion tonight.
copeland
2016-02-26 19:04:02
I'm the school director here at AoPS. I like math.
I'm the school director here at AoPS. I like math.
copeland
2016-02-26 19:04:06
Before we get started I would like to take a moment to explain our virtual classroom procedures to those who have not previously participated in a Math Jam or one of our online classes.
Before we get started I would like to take a moment to explain our virtual classroom procedures to those who have not previously participated in a Math Jam or one of our online classes.
copeland
2016-02-26 19:04:08
The classroom is moderated, meaning that students can type into the classroom, but these comments will not go directly into the room. These comments go to the instructors, who may choose to share your comments with the room.
The classroom is moderated, meaning that students can type into the classroom, but these comments will not go directly into the room. These comments go to the instructors, who may choose to share your comments with the room.
copeland
2016-02-26 19:04:25
This helps keep the class organized and on track. This also means that only well-written comments will be dropped into the classroom, so please take time writing responses that are complete and easy to read.
This helps keep the class organized and on track. This also means that only well-written comments will be dropped into the classroom, so please take time writing responses that are complete and easy to read.
copeland
2016-02-26 19:04:31
There are bunches of students here. As I said, only a fraction of the well-written comments will be passed to the entire group. Please do not take it personally if your comments do not get posted, and please do not complain about it. I expect this Math Jam to be much larger than our typical class, so please be patient with me---there are quite a few of you here tonight!!
There are bunches of students here. As I said, only a fraction of the well-written comments will be passed to the entire group. Please do not take it personally if your comments do not get posted, and please do not complain about it. I expect this Math Jam to be much larger than our typical class, so please be patient with me---there are quite a few of you here tonight!!
copeland
2016-02-26 19:04:39
Also, we won't be going through the math quite as thoroughly as we do in our classes -- I can't teach all the prerequisite material for every problem as we go. Another difference between tonight and our regular online classes is that it is very unlikely that we'll be able to answer every single question you ask. We usually do in our classes, but we have a large number of students tonight! So, please go ahead and ask questions, but also please understand if we aren't able to answer them all!
Also, we won't be going through the math quite as thoroughly as we do in our classes -- I can't teach all the prerequisite material for every problem as we go. Another difference between tonight and our regular online classes is that it is very unlikely that we'll be able to answer every single question you ask. We usually do in our classes, but we have a large number of students tonight! So, please go ahead and ask questions, but also please understand if we aren't able to answer them all!
copeland
2016-02-26 19:04:50
We have 4 (yep, 4!) assistants tonight:
We have 4 (yep, 4!) assistants tonight:
copeland
2016-02-26 19:05:01
Kaitlin Maile (kmaile)
Andrew Jiang (andrewjjiang97)
Ben Stone (hailstone)
Shyan Akmal (Naysh).
Kaitlin Maile (kmaile)
Andrew Jiang (andrewjjiang97)
Ben Stone (hailstone)
Shyan Akmal (Naysh).
copeland
2016-02-26 19:05:11
Kaitlin has been working as a grader for AOPS since November 2014, when she was a senior in high school. While in high school, Kaitlin participated in math team, marching band, and various sports. Now, she is a college student at the University of Minnesota, majoring in Biomedical Engineering. When not working for AOPS or studying for classes, she is usually listening to music of any genre or enjoying the outdoors.
Kaitlin has been working as a grader for AOPS since November 2014, when she was a senior in high school. While in high school, Kaitlin participated in math team, marching band, and various sports. Now, she is a college student at the University of Minnesota, majoring in Biomedical Engineering. When not working for AOPS or studying for classes, she is usually listening to music of any genre or enjoying the outdoors.
copeland
2016-02-26 19:05:17
Shyan is currently a student at Harvey Mudd college, where he plans on majoring in math and computer science. In high school he was an avid participant in science and math competitions, qualifying for the USAMO and USAJMO, being an ARML high scorer two years in a row, and attending the national competitions for Science Bowl and Science Olympiad. His experiences have motivated him to become (more) suspicious of all sciences not based off pure mathematics. Beyond teaching and math, he enjoys reading novels (especially those by Agatha Christie and Terry Pratchett), playing guitar, and ultimate frisbee.
Shyan is currently a student at Harvey Mudd college, where he plans on majoring in math and computer science. In high school he was an avid participant in science and math competitions, qualifying for the USAMO and USAJMO, being an ARML high scorer two years in a row, and attending the national competitions for Science Bowl and Science Olympiad. His experiences have motivated him to become (more) suspicious of all sciences not based off pure mathematics. Beyond teaching and math, he enjoys reading novels (especially those by Agatha Christie and Terry Pratchett), playing guitar, and ultimate frisbee.
copeland
2016-02-26 19:05:18
Ben has a B.S. in Math from Carnegie Mellon University, with minors in Music, Philosophy, and Film Studies, and has an M.S. in Math from the University of Washington. He has spent almost as many hours working on his juggling and unicycling.
Ben has a B.S. in Math from Carnegie Mellon University, with minors in Music, Philosophy, and Film Studies, and has an M.S. in Math from the University of Washington. He has spent almost as many hours working on his juggling and unicycling.
copeland
2016-02-26 19:05:24
And Andrew:
I am currently a freshman at Harvard College, considering concentrating in economics or applied mathematics. In high school, I participated in a lot of math competitions: AMC, AIME, ARML, and the USAMO in 2015. In my free time, I like playing chess, writing and reading poetry, and running. I've also taken some courses on AoPS before including WOOT! A fun fact about me: I type with on a Dvorak-formatted keyboard, instead of the standard QWERTY style.
And Andrew:
I am currently a freshman at Harvard College, considering concentrating in economics or applied mathematics. In high school, I participated in a lot of math competitions: AMC, AIME, ARML, and the USAMO in 2015. In my free time, I like playing chess, writing and reading poetry, and running. I've also taken some courses on AoPS before including WOOT! A fun fact about me: I type with on a Dvorak-formatted keyboard, instead of the standard QWERTY style.
hailstone
2016-02-26 19:05:39
Hello!
Hello!
kmaile32
2016-02-26 19:05:44
Hi!
Hi!
andrewjjiang97
2016-02-26 19:05:47
Hi everyone!
Hi everyone!
Naysh
2016-02-26 19:05:57
Hello.
Hello.
dvdmkr
2016-02-26 19:06:00
hello
hello
Python54
2016-02-26 19:06:00
Hi!
Hi!
suika1234
2016-02-26 19:06:00
hi
hi
kingofmathcounts
2016-02-26 19:06:00
hi
hi
Brainiac2
2016-02-26 19:06:00
hi!
hi!
giraffe123
2016-02-26 19:06:00
Hi!
Hi!
GeneralCobra19
2016-02-26 19:06:00
Hiya!
Hiya!
computernerd
2016-02-26 19:06:00
hi!
hi!
copeland
2016-02-26 19:06:09
We can answer questions by whispering to you or by opening a window with you to chat 1-on-1. However, due to the incredibly large size of the session tonight, they may not be able to get to you right away (or at all). Repeating your question over and over is more likely to annoy us than to get it answered faster, so please, just ask your question once and be patient, and please understand that we may not be able to answer all the questions tonight.
We can answer questions by whispering to you or by opening a window with you to chat 1-on-1. However, due to the incredibly large size of the session tonight, they may not be able to get to you right away (or at all). Repeating your question over and over is more likely to annoy us than to get it answered faster, so please, just ask your question once and be patient, and please understand that we may not be able to answer all the questions tonight.
copeland
2016-02-26 19:06:14
Please also remember that the purpose of this Math Jam is to work through the solutions to AMC problems, and not to merely present the answers. "Working through the solutions" includes discussing problem-solving tactics. So please, when a question is posted, do not simply respond with the final answer. That's not why we're here. We're going to work through the problems step-by-step, and comments that skip key steps or jump ahead in the problem, without providing explanation or motivation, won't be posted.
Please also remember that the purpose of this Math Jam is to work through the solutions to AMC problems, and not to merely present the answers. "Working through the solutions" includes discussing problem-solving tactics. So please, when a question is posted, do not simply respond with the final answer. That's not why we're here. We're going to work through the problems step-by-step, and comments that skip key steps or jump ahead in the problem, without providing explanation or motivation, won't be posted.
copeland
2016-02-26 19:06:18
We will work the last 5 problems from the AMC 10B, then the last 5 problems from the AMC 12B.
We will work the last 5 problems from the AMC 10B, then the last 5 problems from the AMC 12B.
copeland
2016-02-26 19:06:26
Everyone set? Should we get started?
Everyone set? Should we get started?
elitechicken
2016-02-26 19:06:42
yo
yo
Lovemath2015
2016-02-26 19:06:42
hi yah!
hi yah!
nc_sam
2016-02-26 19:06:42
hi
hi
mj7777
2016-02-26 19:06:42
hello
hello
AOPS12142015
2016-02-26 19:06:42
Yes
Yes
avn
2016-02-26 19:06:42
yes
yes
temp8909
2016-02-26 19:06:42
yes!
yes!
Python54
2016-02-26 19:06:42
Yes!
Yes!
Jyzhang12
2016-02-26 19:06:42
YESSS
YESSS
Rotack00
2016-02-26 19:06:42
yes
yes
Brainiac2
2016-02-26 19:06:42
yes!
yes!
abccsparta
2016-02-26 19:06:42
yes
yes
GeneralCobra19
2016-02-26 19:06:42
Yep
Yep
IQMathlete
2016-02-26 19:06:42
yes
yes
math101010
2016-02-26 19:06:42
yeah
yeah
copeland
2016-02-26 19:06:44
OK, let's rock.
OK, let's rock.
temp8909
2016-02-26 19:06:47
will there be any overlap?
will there be any overlap?
copeland
2016-02-26 19:06:57
Strangely, there isn't any overlap in these 10 problems.
Strangely, there isn't any overlap in these 10 problems.
copeland
2016-02-26 19:07:05
Usually there are 1-3 repeats.
Usually there are 1-3 repeats.
copeland
2016-02-26 19:07:27
21. What is the area of the region enclosed by the graph of the equation $x^2 + y^2 = |x| + |y|$?
$\phantom{10B:21}$
(A) $\pi + \sqrt2$ (B) $\pi + 2$ (C) $\pi + 2\sqrt2$ (D) $2\pi + \sqrt2$ (E) $2\pi + 2\sqrt2$
21. What is the area of the region enclosed by the graph of the equation $x^2 + y^2 = |x| + |y|$?
$\phantom{10B:21}$
(A) $\pi + \sqrt2$ (B) $\pi + 2$ (C) $\pi + 2\sqrt2$ (D) $2\pi + \sqrt2$ (E) $2\pi + 2\sqrt2$
copeland
2016-02-26 19:07:38
(This was also #18 on the 12B.)
(This was also #18 on the 12B.)
copeland
2016-02-26 19:07:41
How can we work with those absolute value signs?
How can we work with those absolute value signs?
copeland
2016-02-26 19:08:03
There we go.
There we go.
copeland
2016-02-26 19:08:07
Now, about those absolute values. . .
Now, about those absolute values. . .
MSTang
2016-02-26 19:08:29
Casework
Casework
IQMathlete
2016-02-26 19:08:29
Casework
Casework
physangel
2016-02-26 19:08:29
casework
casework
math101010
2016-02-26 19:08:29
casework
casework
ninjataco
2016-02-26 19:08:29
casework based on signs of x and y
casework based on signs of x and y
temp8909
2016-02-26 19:08:29
divide into cases
divide into cases
SK200
2016-02-26 19:08:29
cases
cases
amyhu910
2016-02-26 19:08:29
casework
casework
copeland
2016-02-26 19:08:33
Casework is a good idea.
Casework is a good idea.
copeland
2016-02-26 19:08:37
What can you say about the cases, though?
What can you say about the cases, though?
joey8189681
2016-02-26 19:09:27
Confine it to one quadrant with symmetry
Confine it to one quadrant with symmetry
Verjok
2016-02-26 19:09:27
Assume that x and y are positive and multiply final answer by 4
Assume that x and y are positive and multiply final answer by 4
SK200
2016-02-26 19:09:27
symmetric
symmetric
bguo
2016-02-26 19:09:27
x is positive results in the same thing as when it is negative
x is positive results in the same thing as when it is negative
ninjataco
2016-02-26 19:09:27
symmetry
symmetry
AkashD
2016-02-26 19:09:27
They are symmetric, so we can focus on one quadrant of the graph
They are symmetric, so we can focus on one quadrant of the graph
DeathLlama9
2016-02-26 19:09:27
They're all the same! (Symmetry)
They're all the same! (Symmetry)
temp8909
2016-02-26 19:09:27
symmetric!!!
symmetric!!!
Benq
2016-02-26 19:09:27
All essentially the same.
All essentially the same.
copeland
2016-02-26 19:09:29
The cases are all symmetric! Replacing $x$ with $-x$ doesn't change anything in the equation. Nor does replacing $y$ with $-y$.
The cases are all symmetric! Replacing $x$ with $-x$ doesn't change anything in the equation. Nor does replacing $y$ with $-y$.
copeland
2016-02-26 19:09:35
The region is the same in each of the four quadrants of the coordinate plane.
The region is the same in each of the four quadrants of the coordinate plane.
copeland
2016-02-26 19:09:37
So we can just compute the region in the first quadrant, where $x \ge 0$ and $y \ge 0$, and then multiply by 4.
So we can just compute the region in the first quadrant, where $x \ge 0$ and $y \ge 0$, and then multiply by 4.
copeland
2016-02-26 19:09:38
In the first quadrant, what do we have?
In the first quadrant, what do we have?
Verjok
2016-02-26 19:10:13
x^2+y^2=x+y
x^2+y^2=x+y
DeathLlama9
2016-02-26 19:10:13
$x^2 + y^2 = x + y$
$x^2 + y^2 = x + y$
Maths4J
2016-02-26 19:10:13
x^2+y^2=x+y
x^2+y^2=x+y
Jyzhang12
2016-02-26 19:10:13
$x^2+y^2=x+y$
$x^2+y^2=x+y$
physangel
2016-02-26 19:10:13
x^2 + y^2=x+y
x^2 + y^2=x+y
speck
2016-02-26 19:10:13
$x^2 + y^2 = x + y$
$x^2 + y^2 = x + y$
copeland
2016-02-26 19:10:16
We just have $x^2 + y^2 = x + y$.
We just have $x^2 + y^2 = x + y$.
copeland
2016-02-26 19:10:16
How can we work with this?
How can we work with this?
GeneralCobra19
2016-02-26 19:10:44
I dunno(completing the square) but it looks like a circle
I dunno(completing the square) but it looks like a circle
bguo
2016-02-26 19:10:44
complete the square
complete the square
MSTang
2016-02-26 19:10:44
Complete the square
Complete the square
ninjataco
2016-02-26 19:10:44
complete the square
complete the square
DeathLlama9
2016-02-26 19:10:44
Subtract $x + y$ from both sides and complete the square
Subtract $x + y$ from both sides and complete the square
suika1234
2016-02-26 19:10:44
complete the square
complete the square
temp8909
2016-02-26 19:10:44
complete the square
complete the square
baseballcat
2016-02-26 19:10:44
complet the square to get a circle
complet the square to get a circle
matthias12345
2016-02-26 19:10:44
make a circle equation
make a circle equation
GeneralCobra19
2016-02-26 19:10:44
Subtract x and y, then complete the square to get the formula of a circle!
Subtract x and y, then complete the square to get the formula of a circle!
ghghghghghghghgh
2016-02-26 19:10:44
complete the square
complete the square
copeland
2016-02-26 19:10:49
We can bring all terms to one side, and complete the square.
We can bring all terms to one side, and complete the square.
copeland
2016-02-26 19:10:51
That is, we have $x^2 - x + y^2 - y = 0$. How do we complete the square from here?
That is, we have $x^2 - x + y^2 - y = 0$. How do we complete the square from here?
swagger21
2016-02-26 19:11:28
i'm having technical problems; the messages aren't sending for me, i have to refresh
i'm having technical problems; the messages aren't sending for me, i have to refresh
baseballcat
2016-02-26 19:11:35
add 1/2 to both sides
add 1/2 to both sides
bguo
2016-02-26 19:11:35
add 1/2 to both sides
add 1/2 to both sides
DeathLlama9
2016-02-26 19:11:35
$(x - \frac{1}{2})^2 + (y - \frac{1}{2})^2 = \frac{1}{2}$
$(x - \frac{1}{2})^2 + (y - \frac{1}{2})^2 = \frac{1}{2}$
idkanymath
2016-02-26 19:11:35
(x-1/2)^2 + (y-1/2)^2=1/2
(x-1/2)^2 + (y-1/2)^2=1/2
GeneralCobra19
2016-02-26 19:11:35
(s-1/2)^2+(y-1/2)^2=1/2
(s-1/2)^2+(y-1/2)^2=1/2
speck
2016-02-26 19:11:35
$\left ( x - \dfrac{1}{2} \right)^2 + \left( x - \dfrac{1}{2} \right)^2 = \dfrac{1}{2}$
$\left ( x - \dfrac{1}{2} \right)^2 + \left( x - \dfrac{1}{2} \right)^2 = \dfrac{1}{2}$
ghghghghghghghgh
2016-02-26 19:11:35
(x-1/2)^2+(y-1/2)^2=1/2
(x-1/2)^2+(y-1/2)^2=1/2
physangel
2016-02-26 19:11:35
add 1/2 to both sides
add 1/2 to both sides
copeland
2016-02-26 19:11:39
We must add $\frac14$ twice to the left side, and thus we add $\frac12$ to the right side:
\[
x^2 - x + \frac14 + y^2 - y + \frac14 = \frac12.
\]
We must add $\frac14$ twice to the left side, and thus we add $\frac12$ to the right side:
\[
x^2 - x + \frac14 + y^2 - y + \frac14 = \frac12.
\]
copeland
2016-02-26 19:11:40
This is just \[ \left(x-\frac12\right)^2 + \left(y-\frac12\right)^2 = \frac12 = \left(\frac{1}{\sqrt2}\right)^2.\]
This is just \[ \left(x-\frac12\right)^2 + \left(y-\frac12\right)^2 = \frac12 = \left(\frac{1}{\sqrt2}\right)^2.\]
copeland
2016-02-26 19:11:41
What does this curve look like?
What does this curve look like?
Verjok
2016-02-26 19:12:03
A circle
A circle
fclvbfm934
2016-02-26 19:12:03
circle
circle
emhyr
2016-02-26 19:12:03
A circle!
A circle!
copeland
2016-02-26 19:12:06
What circle?
What circle?
math101010
2016-02-26 19:12:28
circle with radius sqrt(2)/2 and center (1/2,1/2)
circle with radius sqrt(2)/2 and center (1/2,1/2)
ptes77
2016-02-26 19:12:28
Circle centered at (1/2, 1/2) with a radius of sqrt2/2
Circle centered at (1/2, 1/2) with a radius of sqrt2/2
Caprylic1207
2016-02-26 19:12:28
a circle centered at (1/2,1/2) with radius \sqrt{2}/2
a circle centered at (1/2,1/2) with radius \sqrt{2}/2
DeathLlama9
2016-02-26 19:12:28
It's a circle with radius $\frac{1}{\sqrt{2}}$ centered at $(\frac{1}{2}, \frac{1}{2})$.
It's a circle with radius $\frac{1}{\sqrt{2}}$ centered at $(\frac{1}{2}, \frac{1}{2})$.
baseballcat
2016-02-26 19:12:28
a circle with center at 1/2,1/2 and radius 1/sqrt(2)
a circle with center at 1/2,1/2 and radius 1/sqrt(2)
copeland
2016-02-26 19:12:30
It's a circle centered at $\left(\frac12,\frac12\right)$ and with radius $\frac{1}{\sqrt2}$.
It's a circle centered at $\left(\frac12,\frac12\right)$ and with radius $\frac{1}{\sqrt2}$.
copeland
2016-02-26 19:12:41
copeland
2016-02-26 19:12:44
Except. . .
Except. . .
bguo
2016-02-26 19:13:17
isocels right triangle+semicircle
isocels right triangle+semicircle
DeathLlama9
2016-02-26 19:13:17
Part of it is cut off though
Part of it is cut off though
speck
2016-02-26 19:13:17
First quadrant only
First quadrant only
math101010
2016-02-26 19:13:17
first quadrant
first quadrant
idkanymath
2016-02-26 19:13:17
Only in 1st quadrant
Only in 1st quadrant
GeneralCobra19
2016-02-26 19:13:17
The extras are not on the quadrant
The extras are not on the quadrant
DeathLlama9
2016-02-26 19:13:17
Part of it isn't in the first quadrant
Part of it isn't in the first quadrant
temp8909
2016-02-26 19:13:17
only the part in the first quadrant
only the part in the first quadrant
baseballcat
2016-02-26 19:13:17
we only want it when x and y are positive
we only want it when x and y are positive
IQMathlete
2016-02-26 19:13:17
everything below or to the left of x and y axis
everything below or to the left of x and y axis
copeland
2016-02-26 19:13:23
copeland
2016-02-26 19:13:33
We only want the part in the firs quadrant:
We only want the part in the firs quadrant:
copeland
2016-02-26 19:13:34
copeland
2016-02-26 19:13:37
What's the area of that region?
What's the area of that region?
emhyr
2016-02-26 19:13:59
Typo @copeland
Typo @copeland
copeland
2016-02-26 19:14:08
Sorry, firs is Sweedish for first.
Sorry, firs is Sweedish for first.
phi_ftw1618
2016-02-26 19:14:45
1/2 plus 1/4 pi
1/2 plus 1/4 pi
joey8189681
2016-02-26 19:14:45
pi/4 + 1/2
pi/4 + 1/2
amyhu910
2016-02-26 19:14:45
1/2+pi/4
1/2+pi/4
temp8909
2016-02-26 19:14:45
$\frac{1}{2}+\frac{\pi}{4}$
$\frac{1}{2}+\frac{\pi}{4}$
Verjok
2016-02-26 19:14:45
Semicircle + triangle = $\frac{\pi}{4} + \frac{1}{2}$
Semicircle + triangle = $\frac{\pi}{4} + \frac{1}{2}$
amyhu910
2016-02-26 19:14:45
(1/2)+(pi/4)
(1/2)+(pi/4)
IQMathlete
2016-02-26 19:14:45
1/4 pi + 1/2
1/4 pi + 1/2
emhyr
2016-02-26 19:14:45
$\frac{1}{2}+\frac{\pi}{4}$
$\frac{1}{2}+\frac{\pi}{4}$
Pisensei
2016-02-26 19:14:45
1/4pi + 1/2
1/4pi + 1/2
HVishy
2016-02-26 19:14:45
$1/2+1/4pi$
$1/2+1/4pi$
copeland
2016-02-26 19:14:47
It's a right triangle with legs of length 1, plus half of a circle with radius $\dfrac{1}{\sqrt2}$.
It's a right triangle with legs of length 1, plus half of a circle with radius $\dfrac{1}{\sqrt2}$.
copeland
2016-02-26 19:14:49
Thus, the area is $\dfrac12 + \dfrac14\pi$.
Thus, the area is $\dfrac12 + \dfrac14\pi$.
copeland
2016-02-26 19:14:49
So what's our final answer?
So what's our final answer?
DeathLlama9
2016-02-26 19:15:33
$\pi + 2$
$\pi + 2$
temp8909
2016-02-26 19:15:33
$2+\pi$
$2+\pi$
Caprylic1207
2016-02-26 19:15:33
2+pi
2+pi
Jyzhang12
2016-02-26 19:15:33
$\pi+2$
$\pi+2$
Rotack00
2016-02-26 19:15:33
$2+\pi$
$2+\pi$
mathelp
2016-02-26 19:15:33
pi + 2
pi + 2
AOPS12142015
2016-02-26 19:15:33
2 + pi
2 + pi
speck
2016-02-26 19:15:33
B
B
emhyr
2016-02-26 19:15:33
(B)
(B)
GeneralCobra19
2016-02-26 19:15:33
B
B
Maths4J
2016-02-26 19:15:33
multiply by 4 to get $2+\pi$
multiply by 4 to get $2+\pi$
HVishy
2016-02-26 19:15:33
$\pi+2$
$\pi+2$
SK200
2016-02-26 19:15:33
$\pi + 2$
$\pi + 2$
ninjataco
2016-02-26 19:15:33
pi + 2 (B)
pi + 2 (B)
mathelp
2016-02-26 19:15:33
(B) !!!
(B) !!!
AkashD
2016-02-26 19:15:33
multiply by 4 and get B
multiply by 4 and get B
SineWarrior
2016-02-26 19:15:33
2+pi
2+pi
copeland
2016-02-26 19:15:36
We have four copies of this region (one in each quadrant), so our final answer is 4 times this, or $\boxed{2 + \pi}$. Answer (B).
We have four copies of this region (one in each quadrant), so our final answer is 4 times this, or $\boxed{2 + \pi}$. Answer (B).
copeland
2016-02-26 19:15:36
copeland
2016-02-26 19:15:41
Cool.
Cool.
copeland
2016-02-26 19:15:43
Everyone warmed up?
Everyone warmed up?
temp8909
2016-02-26 19:16:01
yes!
yes!
Firstdream
2016-02-26 19:16:01
Yes
Yes
Rotack00
2016-02-26 19:16:01
yup
yup
DeathLlama9
2016-02-26 19:16:01
Yes!
Yes!
phi_ftw1618
2016-02-26 19:16:01
yeah
yeah
GeneralCobra19
2016-02-26 19:16:01
Of course
Of course
mathelp
2016-02-26 19:16:01
yap
yap
yingyangbu
2016-02-26 19:16:01
ya
ya
Lovemath2015
2016-02-26 19:16:01
yes
yes
Python54
2016-02-26 19:16:01
yes!
yes!
mewtwomew
2016-02-26 19:16:01
yeah!
yeah!
ninjataco
2016-02-26 19:16:01
yesh
yesh
sam15
2016-02-26 19:16:01
yep. Interesting problem
yep. Interesting problem
AOPS12142015
2016-02-26 19:16:01
Yeah!
Yeah!
avn
2016-02-26 19:16:01
yes
yes
physangel
2016-02-26 19:16:01
YES!!!
YES!!!
copeland
2016-02-26 19:16:15
Isn't that the Girl Scouts logo?
Isn't that the Girl Scouts logo?
Lovemath2015
2016-02-26 19:16:34
yeah, the four leaf clover
yeah, the four leaf clover
GeneralCobra19
2016-02-26 19:16:34
is it? not another girl scouts math problem!!!
is it? not another girl scouts math problem!!!
copeland
2016-02-26 19:16:35
I think they want us to eat more cookies.
I think they want us to eat more cookies.
copeland
2016-02-26 19:16:40
We're on to you, AMC.
We're on to you, AMC.
MSTang
2016-02-26 19:16:43
copeland
2016-02-26 19:16:47
OK, so maybe not.
OK, so maybe not.
copeland
2016-02-26 19:16:51
22. A set of teams held a round-robin tournament in which every team played every other team exactly once. Every team won 10 games and lost 10 games; there were no ties. How many sets of three teams $\{A, B, C\}$ were there in which $A$ beat $B$, $B$ beat $C$, and $C$ beat $A$?
$\phantom{10A:22}$
(A) 385 (B) 665 (C) 945 (D) 1140 (E) 1330
22. A set of teams held a round-robin tournament in which every team played every other team exactly once. Every team won 10 games and lost 10 games; there were no ties. How many sets of three teams $\{A, B, C\}$ were there in which $A$ beat $B$, $B$ beat $C$, and $C$ beat $A$?
$\phantom{10A:22}$
(A) 385 (B) 665 (C) 945 (D) 1140 (E) 1330
copeland
2016-02-26 19:17:13
What are you thinking about this one?
What are you thinking about this one?
DeathLlama9
2016-02-26 19:17:39
Complementary counting!
Complementary counting!
baseballcat
2016-02-26 19:17:39
complementary counting
complementary counting
fclvbfm934
2016-02-26 19:17:39
complementary counting
complementary counting
sas4
2016-02-26 19:17:39
complementary counting
complementary counting
suika1234
2016-02-26 19:17:39
complimentary counting
complimentary counting
Superwiz
2016-02-26 19:17:39
Complimentary counting?
Complimentary counting?
physangel
2016-02-26 19:17:39
complementary counting
complementary counting
copeland
2016-02-26 19:17:51
Interesting. That's not the game I like, but let's try it.
Interesting. That's not the game I like, but let's try it.
copeland
2016-02-26 19:18:01
Complementary counting means we count all sets of three teams, and then subtract those that don't satisfy the condition.
Complementary counting means we count all sets of three teams, and then subtract those that don't satisfy the condition.
copeland
2016-02-26 19:18:03
How many sets of three teams are there in total?
How many sets of three teams are there in total?
fclvbfm934
2016-02-26 19:18:35
$\binom{21}{3}$
$\binom{21}{3}$
temp8909
2016-02-26 19:18:35
21 C 3
21 C 3
physangel
2016-02-26 19:18:35
1330
1330
DeathLlama9
2016-02-26 19:18:35
$\binom{21}{3} = 1330$
$\binom{21}{3} = 1330$
idkanymath
2016-02-26 19:18:35
1330
1330
ninjataco
2016-02-26 19:18:35
21C3 = 1330
21C3 = 1330
ghghghghghghghgh
2016-02-26 19:18:35
1330
1330
mathelp
2016-02-26 19:18:35
1330
1330
physangel
2016-02-26 19:18:35
21 choose 3=1330
21 choose 3=1330
AOPS12142015
2016-02-26 19:18:35
21 choose 3
21 choose 3
copeland
2016-02-26 19:18:38
Each team played 20 games and played each other team once. Therefore, there are 21 terms, and hence $\dbinom{21}{3} = 1330$ triples of teams.
Each team played 20 games and played each other team once. Therefore, there are 21 terms, and hence $\dbinom{21}{3} = 1330$ triples of teams.
copeland
2016-02-26 19:18:42
How many of these do not satisfy the condition in the problem?
How many of these do not satisfy the condition in the problem?
copeland
2016-02-26 19:18:47
If $\{A,B,C\}$ don't work, what do we know?
If $\{A,B,C\}$ don't work, what do we know?
DeathLlama9
2016-02-26 19:19:08
One of the teams beat both of the other teams
One of the teams beat both of the other teams
phi_ftw1618
2016-02-26 19:19:08
one team beat the other two
one team beat the other two
fclvbfm934
2016-02-26 19:19:08
One of the players beat the other two
One of the players beat the other two
MSTang
2016-02-26 19:19:08
somebody beat the other two teams
somebody beat the other two teams
baseballcat
2016-02-26 19:19:08
one beat the other 2
one beat the other 2
ghghghghghghghgh
2016-02-26 19:19:08
one of the teams beat both of the others
one of the teams beat both of the others
aq1048576
2016-02-26 19:19:08
one team beat both others in the set
one team beat both others in the set
copeland
2016-02-26 19:19:12
We know that one of the teams must have beaten both of the other two.
We know that one of the teams must have beaten both of the other two.
copeland
2016-02-26 19:19:13
How can we count how often this happens?
How can we count how often this happens?
copeland
2016-02-26 19:19:41
If $A$ is one of the teams, how many $\{B,C\}$ are there such that $A$ beats both $B$ and $C$?
If $A$ is one of the teams, how many $\{B,C\}$ are there such that $A$ beats both $B$ and $C$?
IQMathlete
2016-02-26 19:20:15
for each team, 10c2
for each team, 10c2
sicilianfan
2016-02-26 19:20:15
10C2
10C2
temp8909
2016-02-26 19:20:15
$\binom{10}{2}$
$\binom{10}{2}$
Maths4J
2016-02-26 19:20:15
10C2=45
10C2=45
yingyangbu
2016-02-26 19:20:15
{10 2}
{10 2}
84nea9tresse2
2016-02-26 19:20:15
10c2
10c2
DeathLlama9
2016-02-26 19:20:15
$\binom{10}{2}$
$\binom{10}{2}$
ghghghghghghghgh
2016-02-26 19:20:15
45
45
Peggy
2016-02-26 19:20:15
C (10, 2)
C (10, 2)
copeland
2016-02-26 19:20:17
$A$ beats 10 teams, so $A$ beats $\dbinom{10}{2} = 45$ pairs of teams.
$A$ beats 10 teams, so $A$ beats $\dbinom{10}{2} = 45$ pairs of teams.
copeland
2016-02-26 19:20:19
Thus, for any given $A$, there are 45 sets $\{A,B,C\}$ in which $A$ beats both $B$ and $C$.
Thus, for any given $A$, there are 45 sets $\{A,B,C\}$ in which $A$ beats both $B$ and $C$.
copeland
2016-02-26 19:20:20
So how many such sets in total?
So how many such sets in total?
temp8909
2016-02-26 19:20:56
$21\times 45=945$
$21\times 45=945$
ghghghghghghghgh
2016-02-26 19:20:56
21*45=945
21*45=945
sicilianfan
2016-02-26 19:20:56
21*45
21*45
84nea9tresse2
2016-02-26 19:20:56
45*21
45*21
sam15
2016-02-26 19:20:56
945
945
IQMathlete
2016-02-26 19:20:56
945
945
DeathLlama9
2016-02-26 19:20:56
$21 \times 45 = 945$
$21 \times 45 = 945$
copeland
2016-02-26 19:20:59
There are 21 choices for $A$. So there are $21 \cdot 45 = 945$ sets of three teams that don't work.
There are 21 choices for $A$. So there are $21 \cdot 45 = 945$ sets of three teams that don't work.
copeland
2016-02-26 19:21:03
Thus our final answer, the number of triples that do work, is $1330 - 945 = \boxed{385}$. Answer (A).
Thus our final answer, the number of triples that do work, is $1330 - 945 = \boxed{385}$. Answer (A).
copeland
2016-02-26 19:21:18
Now, did anyone else solve this problem differently?
Now, did anyone else solve this problem differently?
mathelp
2016-02-26 19:21:59
mee
mee
AkashD
2016-02-26 19:21:59
graph theory?
graph theory?
sicilianfan
2016-02-26 19:21:59
graph theory
graph theory
swagger21
2016-02-26 19:22:03
count the number of cases that do work?
count the number of cases that do work?
copeland
2016-02-26 19:22:09
Hm, I LOVE pictures.
Hm, I LOVE pictures.
copeland
2016-02-26 19:22:21
What should we draw?
What should we draw?
Pisensei
2016-02-26 19:22:59
the girl's scout logo
the girl's scout logo
GeneralCobra19
2016-02-26 19:22:59
Another girl scouts logo? Jk
Another girl scouts logo? Jk
copeland
2016-02-26 19:23:06
You guys are killing me with these cookies.
You guys are killing me with these cookies.
copeland
2016-02-26 19:23:15
Let's get to the end and eat some. I assume you have some ready?
Let's get to the end and eat some. I assume you have some ready?
ThorJames
2016-02-26 19:23:21
A triangle with team a and teams that lost and teams that won
A triangle with team a and teams that lost and teams that won
copeland
2016-02-26 19:23:40
Many people are assuming that we want to draw 21 vertices, but that is a lot.
Many people are assuming that we want to draw 21 vertices, but that is a lot.
copeland
2016-02-26 19:23:43
Suppose $A$ is any team. Let $X$ be the teams that $A$ beat, and $Y$ be the teams that beat $A$. There are 10 teams in each set.
Suppose $A$ is any team. Let $X$ be the teams that $A$ beat, and $Y$ be the teams that beat $A$. There are 10 teams in each set.
copeland
2016-02-26 19:23:51
The teams in $X$ and those in $Y$ are similar in some way.
The teams in $X$ and those in $Y$ are similar in some way.
copeland
2016-02-26 19:23:52
We can draw a little diagram to keep track of this, where an arrow points from a winner to a loser.
We can draw a little diagram to keep track of this, where an arrow points from a winner to a loser.
copeland
2016-02-26 19:23:53
copeland
2016-02-26 19:23:54
When do we get a triple $\{A,B,C\}$?
When do we get a triple $\{A,B,C\}$?
phi_ftw1618
2016-02-26 19:24:35
1 in x beats one in y
1 in x beats one in y
GeneralCobra19
2016-02-26 19:24:35
When X beats Y
When X beats Y
SK200
2016-02-26 19:24:35
X beats Y
X beats Y
ghghghghghghghgh
2016-02-26 19:24:35
when a team in X beats a team in Y
when a team in X beats a team in Y
sillyd
2016-02-26 19:24:35
when a team in X beats a team in Y
when a team in X beats a team in Y
amyhu910
2016-02-26 19:24:35
when x beats y?
when x beats y?
copeland
2016-02-26 19:24:36
We need a team in $X$ to beat a team in $Y$.
We need a team in $X$ to beat a team in $Y$.
copeland
2016-02-26 19:24:45
Let's see how many times that happens.
Let's see how many times that happens.
copeland
2016-02-26 19:24:47
Each team in $X$ has 10 wins and 10 losses, so they have a total of 100 wins and 100 losses.
Each team in $X$ has 10 wins and 10 losses, so they have a total of 100 wins and 100 losses.
copeland
2016-02-26 19:25:21
How many of those edges are inside $X$?
How many of those edges are inside $X$?
copeland
2016-02-26 19:26:30
How many wins does X have against X?
How many wins does X have against X?
sicilianfan
2016-02-26 19:26:52
45
45
MSTang
2016-02-26 19:26:52
45
45
Peggy
2016-02-26 19:26:52
45
45
copeland
2016-02-26 19:26:53
We know that they all play each other. That results in $\binom{10}{2} = 45$ wins and 45 losses.
We know that they all play each other. That results in $\binom{10}{2} = 45$ wins and 45 losses.
copeland
2016-02-26 19:26:57
copeland
2016-02-26 19:27:23
So how many times does a team in X beat a team in Y?
So how many times does a team in X beat a team in Y?
avn
2016-02-26 19:27:47
55
55
amyhu910
2016-02-26 19:27:47
55
55
vatatmaja
2016-02-26 19:27:47
55
55
AkashD
2016-02-26 19:27:47
55
55
JayJuly
2016-02-26 19:27:47
55
55
SineWarrior
2016-02-26 19:27:47
55
55
vatatmaja
2016-02-26 19:27:47
100-45
100-45
copeland
2016-02-26 19:27:57
That leaves 55 wins and 45 losses, which must be against the teams in $Y$.
That leaves 55 wins and 45 losses, which must be against the teams in $Y$.
copeland
2016-02-26 19:27:58
copeland
2016-02-26 19:28:03
Again, as a check: there are 100 arrows in total leaving from items in $X$: 45 within $X$, and 55 from $X$ to $Y$. And, there are 100 arrows pointing towards items in $X$: 45 within $X$, 45 from $Y$, and 10 from $A$. (We could do the same check for $Y$ too.)
Again, as a check: there are 100 arrows in total leaving from items in $X$: 45 within $X$, and 55 from $X$ to $Y$. And, there are 100 arrows pointing towards items in $X$: 45 within $X$, 45 from $Y$, and 10 from $A$. (We could do the same check for $Y$ too.)
copeland
2016-02-26 19:28:13
In particular, there are 55 pairs of teams $\{B,C\}$ with $B$ in $X$ and $C$ in $Y$, where $B$ beats $C$. This gives us our set $\{A,B,C\}$.
In particular, there are 55 pairs of teams $\{B,C\}$ with $B$ in $X$ and $C$ in $Y$, where $B$ beats $C$. This gives us our set $\{A,B,C\}$.
copeland
2016-02-26 19:28:16
So since there are 21 choices for $A$, that makes $21 \cdot 55 = 1155$ triples, right?
So since there are 21 choices for $A$, that makes $21 \cdot 55 = 1155$ triples, right?
MSTang
2016-02-26 19:28:49
divide by 3!
divide by 3!
phi_ftw1618
2016-02-26 19:28:49
overcounting
overcounting
W.Sun
2016-02-26 19:28:49
You have to divide by 3
You have to divide by 3
temp8909
2016-02-26 19:28:49
divide by 3
divide by 3
sicilianfan
2016-02-26 19:28:49
triple count
triple count
SK200
2016-02-26 19:28:49
divide by 3
divide by 3
fclvbfm934
2016-02-26 19:28:49
divide by 3
divide by 3
mathelp
2016-02-26 19:28:49
No!! divide by 3
No!! divide by 3
gradysocool
2016-02-26 19:28:49
Over counting by a factor of 3
Over counting by a factor of 3
ninjataco
2016-02-26 19:28:49
no, divide by 3 because overcounting
no, divide by 3 because overcounting
copeland
2016-02-26 19:28:51
No! (That's not an answer choice, for one thing.) Each triple is counted three times, once for each team.
No! (That's not an answer choice, for one thing.) Each triple is counted three times, once for each team.
copeland
2016-02-26 19:28:51
So our final answer is $\dfrac{1155}{3} = \boxed{385}$. Answer (A).
So our final answer is $\dfrac{1155}{3} = \boxed{385}$. Answer (A).
copeland
2016-02-26 19:28:59
Great.
Great.
copeland
2016-02-26 19:29:00
Next problem?
Next problem?
avn
2016-02-26 19:29:24
yup
yup
amyhu910
2016-02-26 19:29:24
bring on the heat!!!!
bring on the heat!!!!
GeneralCobra19
2016-02-26 19:29:24
Sure enough(if it isn't hard)
Sure enough(if it isn't hard)
mathelp
2016-02-26 19:29:24
si
si
SK200
2016-02-26 19:29:24
yeah
yeah
W.Sun
2016-02-26 19:29:24
Yes!
Yes!
W.Sun
2016-02-26 19:29:24
Onward!
Onward!
copeland
2016-02-26 19:29:33
What I liked about that one is that it is independent of the graph.
What I liked about that one is that it is independent of the graph.
copeland
2016-02-26 19:29:41
Not all graphs of this form are isomorphic.
Not all graphs of this form are isomorphic.
copeland
2016-02-26 19:29:46
Anyway. . .
Anyway. . .
copeland
2016-02-26 19:29:47
23. In regular hexagon $ABCDEF$, points $W$, $X$, $Y$, and $Z$ are chosen on sides $\overline{BC}$, $\overline{CD}$, $\overline{EF}$, and $\overline{FA}$, respectively, so that lines $AB$, $ZW$, $YX$, and $ED$ are parallel and equally spaced. What is the ratio of the area of hexagon $WCXYFZ$ to the area of hexagon $ABCDEF$?
$\phantom{10A:23}$
(A) $\dfrac13$ (B) $\dfrac{10}{27}$ (C) $\dfrac{11}{27}$ (D) $\dfrac49$ (E) $\dfrac{13}{27}$.
23. In regular hexagon $ABCDEF$, points $W$, $X$, $Y$, and $Z$ are chosen on sides $\overline{BC}$, $\overline{CD}$, $\overline{EF}$, and $\overline{FA}$, respectively, so that lines $AB$, $ZW$, $YX$, and $ED$ are parallel and equally spaced. What is the ratio of the area of hexagon $WCXYFZ$ to the area of hexagon $ABCDEF$?
$\phantom{10A:23}$
(A) $\dfrac13$ (B) $\dfrac{10}{27}$ (C) $\dfrac{11}{27}$ (D) $\dfrac49$ (E) $\dfrac{13}{27}$.
copeland
2016-02-26 19:29:51
(You might note that the answer choices are consecutive multiples of $\dfrac{1}{27}$. That might help.)
(You might note that the answer choices are consecutive multiples of $\dfrac{1}{27}$. That might help.)
copeland
2016-02-26 19:29:52
I have a feeling that if we just draw an accurate picture, the rest will be easy.
I have a feeling that if we just draw an accurate picture, the rest will be easy.
copeland
2016-02-26 19:29:55
What should we do?
What should we do?
temp8909
2016-02-26 19:30:10
DIAGRAM!!!!
DIAGRAM!!!!
AOPS12142015
2016-02-26 19:30:10
Draw a diagram
Draw a diagram
AlcumusGuy
2016-02-26 19:30:10
draw a picture!
draw a picture!
soccerchess
2016-02-26 19:30:10
draw
draw
vatatmaja
2016-02-26 19:30:10
Draw a Diagram!
Draw a Diagram!
copeland
2016-02-26 19:30:12
copeland
2016-02-26 19:30:15
OK, here's a start.
OK, here's a start.
copeland
2016-02-26 19:30:25
What next? Should we draw more stuff?
What next? Should we draw more stuff?
phi_ftw1618
2016-02-26 19:30:56
yeah, divide the hexagon into 6 equilateral triangles
yeah, divide the hexagon into 6 equilateral triangles
u2d2lrlrba_Ajia
2016-02-26 19:30:56
Would drawing diagonals help?
Would drawing diagonals help?
math101010
2016-02-26 19:30:56
we should draw equilateral triangles
we should draw equilateral triangles
copeland
2016-02-26 19:31:01
We can draw this:
We can draw this:
copeland
2016-02-26 19:31:04
copeland
2016-02-26 19:31:17
In fact, here I see lots of little equilateral triangles.
In fact, here I see lots of little equilateral triangles.
copeland
2016-02-26 19:31:37
One thing we could do is color and then throw math at it:
One thing we could do is color and then throw math at it:
copeland
2016-02-26 19:31:39
copeland
2016-02-26 19:31:47
Is there something simpler we could do?
Is there something simpler we could do?
yrnsmurf
2016-02-26 19:32:20
trisect all edges besides AB and DE
trisect all edges besides AB and DE
AOPS12142015
2016-02-26 19:32:20
Yes, we should draw three lines that are heights since they are equally spaced
Yes, we should draw three lines that are heights since they are equally spaced
phi_ftw1618
2016-02-26 19:32:20
and then divide up those equilateral triangles
and then divide up those equilateral triangles
avn
2016-02-26 19:32:20
split them up more
split them up more
vatatmaja
2016-02-26 19:32:20
Draw a bunch of smaller sized ones!.
Draw a bunch of smaller sized ones!.
copeland
2016-02-26 19:32:32
We can break this up even more, right?
We can break this up even more, right?
copeland
2016-02-26 19:32:46
Let's cut the top half into triangles all of the same size:
Let's cut the top half into triangles all of the same size:
copeland
2016-02-26 19:32:47
copeland
2016-02-26 19:32:58
What's the answer to the problem?
What's the answer to the problem?
phi_ftw1618
2016-02-26 19:33:47
then we can just count them
then we can just count them
math101010
2016-02-26 19:33:47
C
C
sam15
2016-02-26 19:33:47
11/27
11/27
yrnsmurf
2016-02-26 19:33:47
11/27
11/27
kkkittykat
2016-02-26 19:33:47
11/27
11/27
temp8909
2016-02-26 19:33:47
$\frac{11}{27}$
$\frac{11}{27}$
AOPS12142015
2016-02-26 19:33:47
11/27
11/27
ninjataco
2016-02-26 19:33:47
11/27
11/27
Rotack00
2016-02-26 19:33:47
11/27, C
11/27, C
phi_ftw1618
2016-02-26 19:33:47
numerator is clearly 11 so $\frac{11]{27}$
numerator is clearly 11 so $\frac{11]{27}$
sicilianfan
2016-02-26 19:33:47
C
C
yrnsmurf
2016-02-26 19:33:47
$\dfrac{11}{27}$
$\dfrac{11}{27}$
vatatmaja
2016-02-26 19:33:47
C!
C!
vatatmaja
2016-02-26 19:33:47
11/27
11/27
Brainiac2
2016-02-26 19:33:47
c
c
Maths4J
2016-02-26 19:33:47
(B) $\frac{11}{27}$
(B) $\frac{11}{27}$
AC2014
2016-02-26 19:33:47
11/27
11/27
Jayjayliu
2016-02-26 19:33:47
11/27
11/27
jk23541
2016-02-26 19:33:47
C
C
JayJuly
2016-02-26 19:33:47
C
C
W.Sun
2016-02-26 19:33:47
11/27
11/27
W.Sun
2016-02-26 19:33:47
11/27 Is the answer!
11/27 Is the answer!
Math1331Math
2016-02-26 19:33:47
11/27
11/27
IQMathlete
2016-02-26 19:33:47
C
C
mathelp
2016-02-26 19:33:47
11/27!!
11/27!!
sam15
2016-02-26 19:33:47
11/27
11/27
8Invalid8
2016-02-26 19:33:47
11/27!!!!
11/27!!!!
copeland
2016-02-26 19:33:49
Aha! Now we can just count them. There are 11 green triangles and there are 27 total triangles. The ratio is 11/27.
Aha! Now we can just count them. There are 11 green triangles and there are 27 total triangles. The ratio is 11/27.
vatatmaja
2016-02-26 19:33:56
This was a pretty easy 23!
This was a pretty easy 23!
copeland
2016-02-26 19:34:09
One could argue that the problems were not exactly in order on this test.
One could argue that the problems were not exactly in order on this test.
phi_ftw1618
2016-02-26 19:34:32
Would a construction have helped with a more accurate diagram?
Would a construction have helped with a more accurate diagram?
copeland
2016-02-26 19:34:40
So usually that's true, just as a statement.
So usually that's true, just as a statement.
copeland
2016-02-26 19:34:44
I don't think it helps much here.
I don't think it helps much here.
mssmath
2016-02-26 19:35:11
An AoPS person admits the MAA made a mistake? What!!
An AoPS person admits the MAA made a mistake? What!!
copeland
2016-02-26 19:35:12
No. . . I don't think they promised the problems would be in any order.
No. . . I don't think they promised the problems would be in any order.
copeland
2016-02-26 19:35:13
What's next?
What's next?
jk23541
2016-02-26 19:35:21
any tips for being able to see this sort of clever splitting up? Like is there a hint in the problem that gives us this idea?
any tips for being able to see this sort of clever splitting up? Like is there a hint in the problem that gives us this idea?
copeland
2016-02-26 19:35:24
The 27.
The 27.
copeland
2016-02-26 19:35:30
That's all I got.
That's all I got.
copeland
2016-02-26 19:35:38
Plus, it just looks like a graph paper problem.
Plus, it just looks like a graph paper problem.
ThorJames
2016-02-26 19:35:42
24!
24!
copeland
2016-02-26 19:35:45
24. How many four-digit positive integers $abcd$, with $a \not= 0$, have the property that the three two-digit integers $ab < bc < cd$ form an increasing arithmetic sequence? One such number is 4692, where $a=4$, $b=6$, $c=9$, and $d=2$.
$\phantom{10A:24}$
(A) 9 (B) 15 (C) 16 (D) 17 (E) 20
24. How many four-digit positive integers $abcd$, with $a \not= 0$, have the property that the three two-digit integers $ab < bc < cd$ form an increasing arithmetic sequence? One such number is 4692, where $a=4$, $b=6$, $c=9$, and $d=2$.
$\phantom{10A:24}$
(A) 9 (B) 15 (C) 16 (D) 17 (E) 20
copeland
2016-02-26 19:35:49
How can we write this property in a form that we can work with?
How can we write this property in a form that we can work with?
MSTang
2016-02-26 19:36:41
(10a+b) + (10c+d) = 2(10b+c)
(10a+b) + (10c+d) = 2(10b+c)
ninjataco
2016-02-26 19:36:41
10a + b + 10c + d = 20b + 2c
10a + b + 10c + d = 20b + 2c
vatatmaja
2016-02-26 19:36:41
10x+y=xy
10x+y=xy
temp8909
2016-02-26 19:36:41
$10a+b,10b+c,10c+d$
$10a+b,10b+c,10c+d$
AlcumusGuy
2016-02-26 19:36:41
10b + c - (10a + b) = 10c + d - (10b + c)
10b + c - (10a + b) = 10c + d - (10b + c)
IQMathlete
2016-02-26 19:36:41
bc is average of ab and cd
bc is average of ab and cd
Math1331Math
2016-02-26 19:36:41
10a+10c+b+d=20b+2c
10a+10c+b+d=20b+2c
yrnsmurf
2016-02-26 19:36:41
ab+cd=2bc
ab+cd=2bc
copeland
2016-02-26 19:36:43
Note that what they're calling "ab" is really the quantity $10a+b$, etc.
Note that what they're calling "ab" is really the quantity $10a+b$, etc.
copeland
2016-02-26 19:36:45
So the numbers $10a + b$, $10b + c$ and $10c + d$ must be an increasing arithmetic sequence.
So the numbers $10a + b$, $10b + c$ and $10c + d$ must be an increasing arithmetic sequence.
copeland
2016-02-26 19:36:48
Three numbers form an arithmetic sequence if and only if the middle number is the average of the other two.
Three numbers form an arithmetic sequence if and only if the middle number is the average of the other two.
copeland
2016-02-26 19:36:51
In our case, that means that
\[ (10a + b) + (10c + d) = 2(10b + c). \]
In our case, that means that
\[ (10a + b) + (10c + d) = 2(10b + c). \]
copeland
2016-02-26 19:36:55
This simplifies to $10a - 19b + 8c + d = 0$.
This simplifies to $10a - 19b + 8c + d = 0$.
copeland
2016-02-26 19:37:00
How can we work with this to count the solutions?
How can we work with this to count the solutions?
copeland
2016-02-26 19:37:28
What do you notice in that big old expression?
What do you notice in that big old expression?
copeland
2016-02-26 19:38:12
Bee bee bumble bee, I see, um, something that looks like a 2-digit number again.
Bee bee bumble bee, I see, um, something that looks like a 2-digit number again.
vatatmaja
2016-02-26 19:38:40
10a+d=ad!!!
10a+d=ad!!!
ein
2016-02-26 19:38:40
10a+d
10a+d
ThorJames
2016-02-26 19:38:40
10a + d
10a + d
SherlockHolmes7
2016-02-26 19:38:40
10a+d
10a+d
GeneralCobra19
2016-02-26 19:38:40
10a+d!
10a+d!
Rotack00
2016-02-26 19:38:40
10a+d?
10a+d?
copeland
2016-02-26 19:38:50
OK, maybe the 10a+d in there is useful.
OK, maybe the 10a+d in there is useful.
copeland
2016-02-26 19:38:55
We might notice is that $10a + d$ is the two-digit number $\overline{ad}$.
We might notice is that $10a + d$ is the two-digit number $\overline{ad}$.
copeland
2016-02-26 19:38:57
So we might think to isolate it: \[ 10a + d = 19b - 8c. \]
So we might think to isolate it: \[ 10a + d = 19b - 8c. \]
copeland
2016-02-26 19:39:02
At this point, we need to count the number of ways to choose $(b,c)$ to ensure that the right side is a two-digit number that's at least 10 (since $a \not= 0$), and that $ab < bc < cd$.
At this point, we need to count the number of ways to choose $(b,c)$ to ensure that the right side is a two-digit number that's at least 10 (since $a \not= 0$), and that $ab < bc < cd$.
copeland
2016-02-26 19:39:09
Also, the right side only depends on $b$ and $c$. Those are our middle two digits. Is there a more elegant way to express $19b-8c$ that looks like the problem at hand?
Also, the right side only depends on $b$ and $c$. Those are our middle two digits. Is there a more elegant way to express $19b-8c$ that looks like the problem at hand?
copeland
2016-02-26 19:39:59
In particular, can we use the same trick again somehow?
In particular, can we use the same trick again somehow?
MSTang
2016-02-26 19:40:17
(10b+c) + 9(b-c)
(10b+c) + 9(b-c)
AlcumusGuy
2016-02-26 19:40:17
10b + c + 9(b-c)
10b + c + 9(b-c)
copeland
2016-02-26 19:40:23
Ooh, that's interesting.
Ooh, that's interesting.
copeland
2016-02-26 19:40:25
The middle number is $\overline{bc}=10b+c$. We can rewrite our equation as \[10a+d=(10b+c)+9b-9c\]or
The middle number is $\overline{bc}=10b+c$. We can rewrite our equation as \[10a+d=(10b+c)+9b-9c\]or
copeland
2016-02-26 19:40:26
\[\overline{ad}=\overline{bc}-9(c-b).\]
\[\overline{ad}=\overline{bc}-9(c-b).\]
copeland
2016-02-26 19:40:31
As a check, the example they gave us in the problem is $\overline{abcd}=\overline{4692}.$ What does our equation say in this case?
As a check, the example they gave us in the problem is $\overline{abcd}=\overline{4692}.$ What does our equation say in this case?
AlcumusGuy
2016-02-26 19:41:19
42 = 69 - 9(9-6)
42 = 69 - 9(9-6)
ninjataco
2016-02-26 19:41:19
42 = 69 - 9(9-6)
42 = 69 - 9(9-6)
swagger21
2016-02-26 19:41:19
42 = 69 -9(9-6), which is true
42 = 69 -9(9-6), which is true
Math1331Math
2016-02-26 19:41:19
It works
It works
phi_ftw1618
2016-02-26 19:41:19
42=69-27
42=69-27
Dominater76
2016-02-26 19:41:19
42=69-27
42=69-27
Rotack00
2016-02-26 19:41:19
42=69-27
42=69-27
mathmass
2016-02-26 19:41:19
42 = 69 - 9(7)
42 = 69 - 9(7)
yrnsmurf
2016-02-26 19:41:19
42=69-9(9-6)
42=69-9(9-6)
copeland
2016-02-26 19:41:22
The equation says $42=69-9(9-6)$. That's true.
The equation says $42=69-9(9-6)$. That's true.
copeland
2016-02-26 19:41:25
Now we're reduced to finding all the solutions to this equation that
$\bullet$ create four-digit numbers
$\bullet$ create increasing arithmetic sequences.
Now we're reduced to finding all the solutions to this equation that
$\bullet$ create four-digit numbers
$\bullet$ create increasing arithmetic sequences.
copeland
2016-02-26 19:41:27
What do we need to force $\overline{abcd}$ to be a 4-digit number?
What do we need to force $\overline{abcd}$ to be a 4-digit number?
DavidUsa
2016-02-26 19:41:36
What does the bar over ad and bc mean?
What does the bar over ad and bc mean?
copeland
2016-02-26 19:41:46
I'm writing that to mean we put the digits together to make a number.
I'm writing that to mean we put the digits together to make a number.
swagger21
2016-02-26 19:42:01
as opposed to multiplying a*b*c*d
as opposed to multiplying a*b*c*d
copeland
2016-02-26 19:42:02
Exactly.
Exactly.
physangel
2016-02-26 19:42:23
a is not equal to 0
a is not equal to 0
ThorJames
2016-02-26 19:42:23
a not equal 0
a not equal 0
vatatmaja
2016-02-26 19:42:23
a=1,2,3,4...9
a=1,2,3,4...9
Peggy
2016-02-26 19:42:23
a can't be 0
a can't be 0
thebobdbuilder
2016-02-26 19:42:23
0<a
0<a
Rotack00
2016-02-26 19:42:23
a=/=0
a=/=0
sicilianfan
2016-02-26 19:42:23
a=/=0
a=/=0
copeland
2016-02-26 19:42:25
We need $a\geq1$. However, let's stick with thinking about these as two-digit numbers. Then we need $\overline{ad}\geq10$.
We need $a\geq1$. However, let's stick with thinking about these as two-digit numbers. Then we need $\overline{ad}\geq10$.
copeland
2016-02-26 19:42:28
\[10\leq\overline{ad}=\overline{bc}-9(c-b).\]
\[10\leq\overline{ad}=\overline{bc}-9(c-b).\]
copeland
2016-02-26 19:42:37
Now the sequence is guaranteed to be arithmetic because that's where we got the equation. Let's worry about "increasing" along the way.
Now the sequence is guaranteed to be arithmetic because that's where we got the equation. Let's worry about "increasing" along the way.
copeland
2016-02-26 19:42:44
How should we go about finding all the solutions?
How should we go about finding all the solutions?
avn
2016-02-26 19:43:07
casework
casework
Tapeman
2016-02-26 19:43:07
casework
casework
copeland
2016-02-26 19:43:10
Casework on what?
Casework on what?
copeland
2016-02-26 19:44:07
A lot of people are suggesting casework on $a$, but that's a little unsettling. Notice that since $\overline{ad}$ is just any old number, that's EASY. We should focus on the hard constraint.
A lot of people are suggesting casework on $a$, but that's a little unsettling. Notice that since $\overline{ad}$ is just any old number, that's EASY. We should focus on the hard constraint.
Darn
2016-02-26 19:44:24
$c-b$
$c-b$
swagger21
2016-02-26 19:44:24
c-b?
c-b?
copeland
2016-02-26 19:44:31
Let's proceed via casework on $c-b$. First of all, can $c-b$ be negative?
Let's proceed via casework on $c-b$. First of all, can $c-b$ be negative?
Rotack00
2016-02-26 19:45:00
no
no
mathelp
2016-02-26 19:45:00
no
no
Pisensei
2016-02-26 19:45:00
no
no
temp8909
2016-02-26 19:45:00
no
no
swagger21
2016-02-26 19:45:00
no
no
AOPS12142015
2016-02-26 19:45:00
No
No
Maths4J
2016-02-26 19:45:00
no, then cd<bc
no, then cd<bc
ghghghghghghghgh
2016-02-26 19:45:00
no
no
amyhu910
2016-02-26 19:45:00
no?
no?
physangel
2016-02-26 19:45:00
no
no
copeland
2016-02-26 19:45:02
No! If $c<b$ then $\overline{cd}<\overline{bc}$ which contradicts our increasing assumption.
No! If $c<b$ then $\overline{cd}<\overline{bc}$ which contradicts our increasing assumption.
copeland
2016-02-26 19:45:04
Case: $c-b=0$.
Case: $c-b=0$.
copeland
2016-02-26 19:45:04
Then what does our number look like?
Then what does our number look like?
Maths4J
2016-02-26 19:45:30
b=c
b=c
GeneralCobra19
2016-02-26 19:45:30
c=b
c=b
suika1234
2016-02-26 19:45:30
c is equal to b
c is equal to b
copeland
2016-02-26 19:45:31
and. . .
and. . .
temp8909
2016-02-26 19:45:49
bbbb
bbbb
MSTang
2016-02-26 19:45:49
ad = bc, so we would have a number like bbbb
ad = bc, so we would have a number like bbbb
SK200
2016-02-26 19:45:49
same digiy
same digiy
PwnageBeans
2016-02-26 19:45:49
a=b=c=d?
a=b=c=d?
Tapeman
2016-02-26 19:45:49
and a=b=c=d
and a=b=c=d
copeland
2016-02-26 19:45:53
Our equation reads $\overline{ad}=\overline{bb}$, so $a=d$ as well and the number is $aaaa$. This gives an arithmetic sequence but it is not increasing.
Our equation reads $\overline{ad}=\overline{bb}$, so $a=d$ as well and the number is $aaaa$. This gives an arithmetic sequence but it is not increasing.
copeland
2016-02-26 19:45:58
There are no solutions with the two middle number equal.
There are no solutions with the two middle number equal.
copeland
2016-02-26 19:45:59
Case: $c-b=1$
Case: $c-b=1$
copeland
2016-02-26 19:46:00
The equation reads $\overline{ad}=\overline{bc}-9$ with $c=b+1$. So $\overline{bc}$ is one of 12, 23, 34, 45, 56, 67, 78, or 89.
The equation reads $\overline{ad}=\overline{bc}-9$ with $c=b+1$. So $\overline{bc}$ is one of 12, 23, 34, 45, 56, 67, 78, or 89.
copeland
2016-02-26 19:46:06
Which don't work?
Which don't work?
vatatmaja
2016-02-26 19:46:50
12
12
baseballcat
2016-02-26 19:46:50
12
12
gradysocool
2016-02-26 19:46:50
12
12
ghghghghghghghgh
2016-02-26 19:46:50
12
12
physangel
2016-02-26 19:46:50
12
12
soccerchess
2016-02-26 19:46:50
12
12
Peggy
2016-02-26 19:46:50
12
12
copeland
2016-02-26 19:46:53
If $\overline{bc}=12$ then $\overline{ad}=12-9<10$ so that fails. Otherwise we get solutions:\[
1234\quad
2345\quad
3456\quad
4567\quad
5678\quad
6789\quad
8890.\]
If $\overline{bc}=12$ then $\overline{ad}=12-9<10$ so that fails. Otherwise we get solutions:\[
1234\quad
2345\quad
3456\quad
4567\quad
5678\quad
6789\quad
8890.\]
copeland
2016-02-26 19:46:54
(Note that 8890 is one that a lot of people miss in most approaches to this problem!)
(Note that 8890 is one that a lot of people miss in most approaches to this problem!)
copeland
2016-02-26 19:47:01
We had to eliminate 0123 from the list because we failed the inequality $10\leq\overline{bc}-9(c-b)$.
We had to eliminate 0123 from the list because we failed the inequality $10\leq\overline{bc}-9(c-b)$.
copeland
2016-02-26 19:47:03
There are 7 values in this case.
There are 7 values in this case.
copeland
2016-02-26 19:47:04
Case: $c-b=2$
Case: $c-b=2$
copeland
2016-02-26 19:47:06
What is the smallest $\overline{bc}$ that solves $10\leq \overline{bc}-9(c-b)$ now?
What is the smallest $\overline{bc}$ that solves $10\leq \overline{bc}-9(c-b)$ now?
ghghghghghghghgh
2016-02-26 19:47:51
35
35
MSTang
2016-02-26 19:47:51
35
35
yrnsmurf
2016-02-26 19:47:51
35
35
Pisensei
2016-02-26 19:47:51
35
35
Darn
2016-02-26 19:47:51
$35$
$35$
Jayjayliu
2016-02-26 19:47:51
35
35
Rotack00
2016-02-26 19:47:51
35
35
copeland
2016-02-26 19:47:54
We want $10\leq\overline{bc}-18,$ so $28\leq\overline{bc}$. The first number that solves this is 35. How many valid 4-digit numbers are in this case?
We want $10\leq\overline{bc}-18,$ so $28\leq\overline{bc}$. The first number that solves this is 35. How many valid 4-digit numbers are in this case?
copeland
2016-02-26 19:48:04
The possible values for $\overline{bc}$ are 35, 46, 57, 68, 79. There are 5 solutions\[
1357\quad
2468\quad
3579\quad
5680\quad
6791.\]
The possible values for $\overline{bc}$ are 35, 46, 57, 68, 79. There are 5 solutions\[
1357\quad
2468\quad
3579\quad
5680\quad
6791.\]
copeland
2016-02-26 19:48:10
Case: $c-b=3$
Case: $c-b=3$
copeland
2016-02-26 19:48:10
What are all possible values of $\overline{bc}$ in this case?
What are all possible values of $\overline{bc}$ in this case?
gradysocool
2016-02-26 19:48:23
bash out the other cases now?
bash out the other cases now?
copeland
2016-02-26 19:48:26
Yes. . .
Yes. . .
Darn
2016-02-26 19:48:55
$47, 58, 69$
$47, 58, 69$
temp8909
2016-02-26 19:48:55
47,58,69
47,58,69
yrnsmurf
2016-02-26 19:48:55
47, 58, 69
47, 58, 69
ghghghghghghghgh
2016-02-26 19:48:55
47,58,69
47,58,69
ninjataco
2016-02-26 19:48:55
47, 58, 69
47, 58, 69
Pisensei
2016-02-26 19:48:55
47, 58, 69
47, 58, 69
Jayjayliu
2016-02-26 19:48:55
47, 58, 69
47, 58, 69
copeland
2016-02-26 19:48:57
We need $10\leq\overline{bc}-27$ or $\overline{bc}\geq37$. The possible values for $\overline{bc}$ are 47, 58, 69.
We need $10\leq\overline{bc}-27$ or $\overline{bc}\geq37$. The possible values for $\overline{bc}$ are 47, 58, 69.
copeland
2016-02-26 19:48:59
There are 3 solutions:\[
2470\quad
3581\quad
4692.\]
There are 3 solutions:\[
2470\quad
3581\quad
4692.\]
Dominater76
2016-02-26 19:49:02
Well that's kinds boring /:
Well that's kinds boring /:
copeland
2016-02-26 19:49:03
Yes.
Yes.
copeland
2016-02-26 19:49:07
Hopefully we're almost done.
Hopefully we're almost done.
copeland
2016-02-26 19:49:08
Case: $c-b=4$
Case: $c-b=4$
copeland
2016-02-26 19:49:09
What are the solutions now?
What are the solutions now?
yrnsmurf
2016-02-26 19:49:58
48 and 59
48 and 59
swagger21
2016-02-26 19:49:58
48, 59
48, 59
Pisensei
2016-02-26 19:49:58
48, 59
48, 59
AOPS12142015
2016-02-26 19:49:58
48,59
48,59
ghghghghghghghgh
2016-02-26 19:49:58
48,59
48,59
Darn
2016-02-26 19:49:58
$48, 59$
$48, 59$
Brainiac2
2016-02-26 19:49:58
59,48
59,48
copeland
2016-02-26 19:50:02
We need $10\leq\overline{bc}-36$ or $\overline{bc}\geq47$. The only possible values for $\overline{bc}$ are 48 and 59.
We need $10\leq\overline{bc}-36$ or $\overline{bc}\geq47$. The only possible values for $\overline{bc}$ are 48 and 59.
copeland
2016-02-26 19:50:03
So there are two solutions: \[ 1482 \quad 2593.\]
So there are two solutions: \[ 1482 \quad 2593.\]
copeland
2016-02-26 19:50:04
Are there any other cases?
Are there any other cases?
avn
2016-02-26 19:50:28
no
no
agbdmrbirdyface
2016-02-26 19:50:28
no
no
MSTang
2016-02-26 19:50:28
no
no
Darn
2016-02-26 19:50:28
No, since $49$ doesn't work.
No, since $49$ doesn't work.
ghghghghghghghgh
2016-02-26 19:50:28
no
no
Rotack00
2016-02-26 19:50:28
no?
no?
math101010
2016-02-26 19:50:28
no
no
yrnsmurf
2016-02-26 19:50:28
nope
nope
PwnageBeans
2016-02-26 19:50:28
no
no
lkarhat
2016-02-26 19:50:28
no
no
AOPS12142015
2016-02-26 19:50:28
no
no
copeland
2016-02-26 19:50:32
There are no other cases: If $c-b\geq5$ then $\overline{bc}\geq10+45,$ but then $b\geq6$ so $c\geq11$ and that's not how digits work.
There are no other cases: If $c-b\geq5$ then $\overline{bc}\geq10+45,$ but then $b\geq6$ so $c\geq11$ and that's not how digits work.
copeland
2016-02-26 19:50:38
So what's the answer?
So what's the answer?
kkkittykat
2016-02-26 19:51:04
17
17
avn
2016-02-26 19:51:04
17
17
jeremyyu
2016-02-26 19:51:04
17
17
Rotack00
2016-02-26 19:51:04
17
17
nosaj
2016-02-26 19:51:04
17!
17!
AOPS12142015
2016-02-26 19:51:04
17 (D)
17 (D)
jeremyyu
2016-02-26 19:51:04
17 D
17 D
mathelp
2016-02-26 19:51:04
D (17)
D (17)
avn
2016-02-26 19:51:04
d
d
physangel
2016-02-26 19:51:04
D,17
D,17
Darn
2016-02-26 19:51:04
$7+5+3+2=17$.
$7+5+3+2=17$.
SineWarrior
2016-02-26 19:51:04
17
17
avn
2016-02-26 19:51:04
17
17
ninjataco
2016-02-26 19:51:04
17
17
Pisensei
2016-02-26 19:51:04
17
17
copeland
2016-02-26 19:51:06
There are $7+5+3+2=\boxed{17}$ solutions. The answer is D.
There are $7+5+3+2=\boxed{17}$ solutions. The answer is D.
copeland
2016-02-26 19:51:35
Alright, time to finish off this test so we can move to the next one.
Alright, time to finish off this test so we can move to the next one.
mathelp
2016-02-26 19:51:43
YAY 25 now!!
YAY 25 now!!
temp8909
2016-02-26 19:51:43
next one!
next one!
Darn
2016-02-26 19:51:45
That took a while.
That took a while.
copeland
2016-02-26 19:51:49
Yeah, 24 was pretty hard.
Yeah, 24 was pretty hard.
Maths4J
2016-02-26 19:51:55
I couldn't follow this complex casework. Isn't there a better way?
I couldn't follow this complex casework. Isn't there a better way?
copeland
2016-02-26 19:51:56
No.
No.
copeland
2016-02-26 19:52:01
There are a lot of different things you can do.
There are a lot of different things you can do.
copeland
2016-02-26 19:52:10
In a different approach, I also got to this cool equation:
In a different approach, I also got to this cool equation:
copeland
2016-02-26 19:52:22
\[10(a-2b+c)=-(b-2c+d).\]
\[10(a-2b+c)=-(b-2c+d).\]
copeland
2016-02-26 19:52:30
Now if those are digits there aren't a lot of solutions.
Now if those are digits there aren't a lot of solutions.
copeland
2016-02-26 19:52:40
Since $-18<-(b-2c+d)<18,$ we know $a-2b+c$ is -1, 0, or 1.
Since $-18<-(b-2c+d)<18,$ we know $a-2b+c$ is -1, 0, or 1.
copeland
2016-02-26 19:53:12
For example, if those are both zero we have \[a-2b+c=0\]and\[b-2c+d=0.\]
For example, if those are both zero we have \[a-2b+c=0\]and\[b-2c+d=0.\]
copeland
2016-02-26 19:53:20
That means the numbers are an honest arithmetic progression.
That means the numbers are an honest arithmetic progression.
copeland
2016-02-26 19:53:29
Unfortunately the other 2 cases are still pretty annoying.
Unfortunately the other 2 cases are still pretty annoying.
SineWarrior
2016-02-26 19:53:39
where did the 18 come from
where did the 18 come from
copeland
2016-02-26 19:53:51
20-2=18.
20-2=18.
copeland
2016-02-26 19:53:59
Oh, no. that's not right.
Oh, no. that's not right.
copeland
2016-02-26 19:54:02
9+9=18.
9+9=18.
copeland
2016-02-26 19:54:15
And 2*9 is also 18.
And 2*9 is also 18.
MSTang
2016-02-26 19:54:26
You could just stick with the initial equation (10a-19b+8c-d=0) and bash, right?
You could just stick with the initial equation (10a-19b+8c-d=0) and bash, right?
copeland
2016-02-26 19:54:35
Probably, but when I tried it I missed all kinds of answers.
Probably, but when I tried it I missed all kinds of answers.
copeland
2016-02-26 19:54:43
Problem 25:
Problem 25:
copeland
2016-02-26 19:54:47
25. Let $f(x) = \sum_{k=2}^{10}\left(\lfloor kx \rfloor - k \lfloor x \rfloor\right)$, where $\lfloor r \rfloor$ denotes the greatest integer less than or equal to $r$. How many distinct values does $f(x)$ assume for $x \ge 0$?
$\phantom{10A:25}$
(A) 32 (B) 36 (C) 45 (D) 46 (E) infinitely many
25. Let $f(x) = \sum_{k=2}^{10}\left(\lfloor kx \rfloor - k \lfloor x \rfloor\right)$, where $\lfloor r \rfloor$ denotes the greatest integer less than or equal to $r$. How many distinct values does $f(x)$ assume for $x \ge 0$?
$\phantom{10A:25}$
(A) 32 (B) 36 (C) 45 (D) 46 (E) infinitely many
copeland
2016-02-26 19:54:58
How can we deal with the floor function?
How can we deal with the floor function?
math101010
2016-02-26 19:55:26
casework
casework
legozelda
2016-02-26 19:55:26
casework?
casework?
copeland
2016-02-26 19:55:27
Who's trolling whom?
Who's trolling whom?
swagger21
2016-02-26 19:55:48
MAA's trolling us
MAA's trolling us
GeneralCobra19
2016-02-26 19:55:48
The MAA is trolling us
The MAA is trolling us
gradysocool
2016-02-26 19:56:00
separate into integer and fractional parts
separate into integer and fractional parts
AlcumusGuy
2016-02-26 19:56:00
split it into the number minus its fractional part
split it into the number minus its fractional part
mathguy5041
2016-02-26 19:56:00
write x as integer part + decimal part
write x as integer part + decimal part
Darn
2016-02-26 19:56:00
Let it equal $n-\{n\}$, where $\{n\}$ is the fractional part of $n$.
Let it equal $n-\{n\}$, where $\{n\}$ is the fractional part of $n$.
nosaj
2016-02-26 19:56:00
x - fractional part of x
x - fractional part of x
Puzzled417
2016-02-26 19:56:00
let x = m+r where r is the remainder 0 < r < 1
let x = m+r where r is the remainder 0 < r < 1
copeland
2016-02-26 19:56:04
One convenient way is to split a number into its integer part and its fractional part.
One convenient way is to split a number into its integer part and its fractional part.
copeland
2016-02-26 19:56:05
That is, for any $x$, we write $x = y + z$, where $y$ is an integer and $0 \le z < 1$. Note that $y = \lfloor x \rfloor$ and we sometimes write $z = \{ x \}$.
That is, for any $x$, we write $x = y + z$, where $y$ is an integer and $0 \le z < 1$. Note that $y = \lfloor x \rfloor$ and we sometimes write $z = \{ x \}$.
copeland
2016-02-26 19:56:07
Then what happens to the expression inside the sum?
Then what happens to the expression inside the sum?
copeland
2016-02-26 19:57:49
OK, so first we know $\lfloor x\rfloor =y$, so we get $\lfloor kx \rfloor - k \lfloor x \rfloor = \lfloor k(y+z) \rfloor - ky$.
OK, so first we know $\lfloor x\rfloor =y$, so we get $\lfloor kx \rfloor - k \lfloor x \rfloor = \lfloor k(y+z) \rfloor - ky$.
copeland
2016-02-26 19:57:51
But then what?
But then what?
nosaj
2016-02-26 19:59:08
it becomes $k\{x\}$
it becomes $k\{x\}$
yrnsmurf
2016-02-26 19:59:08
it beomes floor(kz)
it beomes floor(kz)
MSTang
2016-02-26 19:59:08
take out the ky from the floor
take out the ky from the floor
Dominater76
2016-02-26 19:59:08
ky cancels
ky cancels
Hyperspace01
2016-02-26 19:59:08
$ky-ky+\lfloor kz\rfloor=\lfloor kz\rfloor$
$ky-ky+\lfloor kz\rfloor=\lfloor kz\rfloor$
AlcumusGuy
2016-02-26 19:59:08
floor(k(y+z)) = floor(ky + kz) = ky + floor(kz)
floor(k(y+z)) = floor(ky + kz) = ky + floor(kz)
sillyd
2016-02-26 19:59:08
ky is an integer so just take it out of the floor function
ky is an integer so just take it out of the floor function
ghghghghghghghgh
2016-02-26 19:59:08
take out ky from the floor
take out ky from the floor
randomdude10807
2016-02-26 19:59:08
it is floor(kz) i think
it is floor(kz) i think
copeland
2016-02-26 19:59:16
$ky$ is an integer, so we can pull it out of the floor function: \[ \lfloor k(y+z) \rfloor = ky + \lfloor kz \rfloor.\]
$ky$ is an integer, so we can pull it out of the floor function: \[ \lfloor k(y+z) \rfloor = ky + \lfloor kz \rfloor.\]
copeland
2016-02-26 19:59:18
Now the $ky$ parts cancel, and we're left with $\lfloor kx \rfloor - k \lfloor x \rfloor = \lfloor kz \rfloor$.
Now the $ky$ parts cancel, and we're left with $\lfloor kx \rfloor - k \lfloor x \rfloor = \lfloor kz \rfloor$.
copeland
2016-02-26 19:59:20
That means that \[f(x) = \sum_{k=2}^{10} \lfloor kz \rfloor,\] where $0 \le z < 1$ is the fractional part of $x$.
That means that \[f(x) = \sum_{k=2}^{10} \lfloor kz \rfloor,\] where $0 \le z < 1$ is the fractional part of $x$.
copeland
2016-02-26 19:59:23
That's great, because now we can forget about $x$ and just worry about $0 \le z < 1$. In fact, I'm going to call it $f(z)$.
That's great, because now we can forget about $x$ and just worry about $0 \le z < 1$. In fact, I'm going to call it $f(z)$.
copeland
2016-02-26 19:59:25
What do we know about $f(z)$?
What do we know about $f(z)$?
copeland
2016-02-26 19:59:37
Let's look at each term of the sum separately. What about the $k=2$ term?
Let's look at each term of the sum separately. What about the $k=2$ term?
temp8909
2016-02-26 20:00:17
0 or 1
0 or 1
randomdude10807
2016-02-26 20:00:17
f(z) is 0 or 1
f(z) is 0 or 1
copeland
2016-02-26 20:00:18
When is it 0 and when is it 1?
When is it 0 and when is it 1?
agbdmrbirdyface
2016-02-26 20:00:45
it depends on the value of z: if z > 1/2 we have 1, and if z < 1/2 we have 0
it depends on the value of z: if z > 1/2 we have 1, and if z < 1/2 we have 0
epiclucario
2016-02-26 20:00:45
if z<.5, the term is 0, if z>=.5, the term is 1
if z<.5, the term is 0, if z>=.5, the term is 1
mathguy5041
2016-02-26 20:00:45
x<1/2: 0
x<1/2: 0
gradysocool
2016-02-26 20:00:45
0 below 1/2 1 above 1/2
0 below 1/2 1 above 1/2
WhaleVomit
2016-02-26 20:00:45
0 when z<0.5
0 when z<0.5
agbdmrbirdyface
2016-02-26 20:00:45
0 when z < 1/2, 1 when z >= 1/2
0 when z < 1/2, 1 when z >= 1/2
copeland
2016-02-26 20:00:48
It's 0 for $0 \le z < \frac12$ and it's 1 for $\frac12 \le z < 1$.
It's 0 for $0 \le z < \frac12$ and it's 1 for $\frac12 \le z < 1$.
copeland
2016-02-26 20:00:48
What about the $k=3$ term?
What about the $k=3$ term?
mathguy5041
2016-02-26 20:01:35
split into thirds
split into thirds
gradysocool
2016-02-26 20:01:35
0,1,2 between the thirds
0,1,2 between the thirds
soccerchess
2016-02-26 20:01:35
0,1,2
0,1,2
Burrito
2016-02-26 20:01:35
it is 0 or 1 or 2
it is 0 or 1 or 2
ninjataco
2016-02-26 20:01:35
0 when 0<= z < 1/3, 1 when 1/3 <= z < 2/3, 2 when 2/3 <= z < 1
0 when 0<= z < 1/3, 1 when 1/3 <= z < 2/3, 2 when 2/3 <= z < 1
AlcumusGuy
2016-02-26 20:01:35
0 if $0 \le z < \frac{1}{3}$, 1 if $\frac{1}{3} \le z < \frac{2}{3}$, and 2 otherwise
0 if $0 \le z < \frac{1}{3}$, 1 if $\frac{1}{3} \le z < \frac{2}{3}$, and 2 otherwise
agbdmrbirdyface
2016-02-26 20:01:35
0 for when z < 1/3, 1 when z is between 1/3 and 2/3, and 2 for z above 2/3
0 for when z < 1/3, 1 when z is between 1/3 and 2/3, and 2 for z above 2/3
swagger21
2016-02-26 20:01:35
0 when z < 1/3, 1 when 1/3 ≤ z < 2/3, 2 when 2/3 ≤ z < 1
0 when z < 1/3, 1 when 1/3 ≤ z < 2/3, 2 when 2/3 ≤ z < 1
copeland
2016-02-26 20:01:37
It's 0 for $0 \le z < \frac13$, it's 1 for $\frac13 \le z < \frac23$, and it's 2 for $\frac23 \le z < 1$.
It's 0 for $0 \le z < \frac13$, it's 1 for $\frac13 \le z < \frac23$, and it's 2 for $\frac23 \le z < 1$.
copeland
2016-02-26 20:01:39
See the pattern? The $k^\text{th}$ term increases by 1 whenever $z$ crosses a fraction $\frac{m}{k}$ for some $0 < m < k$.
See the pattern? The $k^\text{th}$ term increases by 1 whenever $z$ crosses a fraction $\frac{m}{k}$ for some $0 < m < k$.
copeland
2016-02-26 20:01:40
So the $k=2$ terms "steps up" once, the $k=3$ term "steps up" twice, etc., up to the $k=10$ term "steps up" 9 times.
So the $k=2$ terms "steps up" once, the $k=3$ term "steps up" twice, etc., up to the $k=10$ term "steps up" 9 times.
copeland
2016-02-26 20:01:41
So, since we start with $f(0) = 0$ and we step up $1+2+\cdots+9 = 45$ times, is the answer 46?
So, since we start with $f(0) = 0$ and we step up $1+2+\cdots+9 = 45$ times, is the answer 46?
Darn
2016-02-26 20:02:29
No, because you repeat some fractions.
No, because you repeat some fractions.
agbdmrbirdyface
2016-02-26 20:02:29
like some cases for k = 4 and k = 2 overlap, etc.
like some cases for k = 4 and k = 2 overlap, etc.
AlcumusGuy
2016-02-26 20:02:29
no, because some fractions are overcounted; for example $\frac{3}{6} = \frac{1}{2}$
no, because some fractions are overcounted; for example $\frac{3}{6} = \frac{1}{2}$
copeland
2016-02-26 20:02:31
No! Some of these "steps" overlap.
No! Some of these "steps" overlap.
copeland
2016-02-26 20:02:32
For example, at $z=\frac14$, we step up the $k=4$ term, but we also step up the $k=8$ term (since $\frac14 = \frac28$). That's a total "step up" of 2, but we only want to count it once because it only produces one new value.
For example, at $z=\frac14$, we step up the $k=4$ term, but we also step up the $k=8$ term (since $\frac14 = \frac28$). That's a total "step up" of 2, but we only want to count it once because it only produces one new value.
copeland
2016-02-26 20:02:34
So how to we count the "step ups" so that we don't overcount the overlaps?
So how to we count the "step ups" so that we don't overcount the overlaps?
swagger21
2016-02-26 20:03:41
count how many distinct reduced proper fractions with denominator ≤ 10 there are
count how many distinct reduced proper fractions with denominator ≤ 10 there are
copeland
2016-02-26 20:03:46
Yeah, OK.
Yeah, OK.
copeland
2016-02-26 20:03:50
One way is to note that each new $k$ only produces a new "step up" at $\dfrac{m}{k}$ if $m$ is relatively prime to $m$; that is, if $\dfrac{m}{k}$ is in lowest terms. Because, if $\dfrac{m}{k}$ can be reduced to a smaller denominator, then we've already counted that "step up" when we counted the smaller denominator.
One way is to note that each new $k$ only produces a new "step up" at $\dfrac{m}{k}$ if $m$ is relatively prime to $m$; that is, if $\dfrac{m}{k}$ is in lowest terms. Because, if $\dfrac{m}{k}$ can be reduced to a smaller denominator, then we've already counted that "step up" when we counted the smaller denominator.
idomath12345
2016-02-26 20:03:54
Bash the whole set out.
Bash the whole set out.
copeland
2016-02-26 20:04:04
Ayup. I might just start writing them down now.
Ayup. I might just start writing them down now.
copeland
2016-02-26 20:04:12
In fact. . .
In fact. . .
copeland
2016-02-26 20:04:14
\[\begin{array}{r|l|l}
k & \text{fractions} & \text{number} \\ \hline
2 & \frac12 & 1 \\
3 & \frac13,\frac23 & 2 \\
4 & \frac14,\frac34 & 2 \\
5 & \frac15,\frac25,\frac35,\frac45 & 4 \\
6 & \frac16,\frac56 & 2 \\
7 & \frac17,\frac27,\frac37,\frac47,\frac57,\frac67 & 6 \\
8 & \frac18,\frac38,\frac58,\frac78 & 4 \\
9 & \frac19,\frac29,\frac49,\frac59,\frac79,\frac89 & 6 \\
10 & \frac{1}{10},\frac{3}{10},\frac{7}{10},\frac{9}{10} & 4
\end{array}\]
\[\begin{array}{r|l|l}
k & \text{fractions} & \text{number} \\ \hline
2 & \frac12 & 1 \\
3 & \frac13,\frac23 & 2 \\
4 & \frac14,\frac34 & 2 \\
5 & \frac15,\frac25,\frac35,\frac45 & 4 \\
6 & \frac16,\frac56 & 2 \\
7 & \frac17,\frac27,\frac37,\frac47,\frac57,\frac67 & 6 \\
8 & \frac18,\frac38,\frac58,\frac78 & 4 \\
9 & \frac19,\frac29,\frac49,\frac59,\frac79,\frac89 & 6 \\
10 & \frac{1}{10},\frac{3}{10},\frac{7}{10},\frac{9}{10} & 4
\end{array}\]
copeland
2016-02-26 20:04:17
Ta-da!
Ta-da!
Darn
2016-02-26 20:04:23
The number of common fractions with denominator $n$ is equal to $\phi(n)$, so we can sum $\phi(1)+\phi(2)+\ldots+\phi(10)$.
The number of common fractions with denominator $n$ is equal to $\phi(n)$, so we can sum $\phi(1)+\phi(2)+\ldots+\phi(10)$.
copeland
2016-02-26 20:04:30
Fancy.
Fancy.
copeland
2016-02-26 20:04:34
(If you know the Euler $\phi$-function, then you might be able to do this more quickly.)
(If you know the Euler $\phi$-function, then you might be able to do this more quickly.)
copeland
2016-02-26 20:04:43
That's a total of 1+2+2+4+2+6+4+6+4 = 31 "step ups" on the way from $z=0$ to $z=1$. Each "step up" increases the value of $f(z)$.
That's a total of 1+2+2+4+2+6+4+6+4 = 31 "step ups" on the way from $z=0$ to $z=1$. Each "step up" increases the value of $f(z)$.
greenguy03
2016-02-26 20:04:51
1+2+2+4+2+6+4+6+4=31
1+2+2+4+2+6+4+6+4=31
agbdmrbirdyface
2016-02-26 20:04:51
which yields 31
which yields 31
copeland
2016-02-26 20:04:55
Since we start with $f(z) = 0$, and step it up 31 times on the way to $z=1$, we get a total of $\boxed{32}$ different values. Answer (A).
Since we start with $f(z) = 0$, and step it up 31 times on the way to $z=1$, we get a total of $\boxed{32}$ different values. Answer (A).
copeland
2016-02-26 20:05:20
Cool.
Cool.
copeland
2016-02-26 20:05:25
The 10 is down.
The 10 is down.
copeland
2016-02-26 20:05:33
30 points us. Go us!
30 points us. Go us!
copeland
2016-02-26 20:05:42
Should we go get 30 more points?
Should we go get 30 more points?
temp8909
2016-02-26 20:06:04
on to 12!
on to 12!
avn
2016-02-26 20:06:04
now the 12
now the 12
Aathreyakadambi
2016-02-26 20:06:04
temp8909
2016-02-26 20:06:04
from the 12!
from the 12!
Tapeman
2016-02-26 20:06:04
yes
yes
darthvader1521
2016-02-26 20:06:04
yes
yes
ThorJames
2016-02-26 20:06:04
yes!
yes!
weeshree
2016-02-26 20:06:04
sure!
sure!
yrnsmurf
2016-02-26 20:06:04
definetly
definetly
Magikarp1
2016-02-26 20:06:04
yes
yes
mj7777
2016-02-26 20:06:04
yes
yes
baseballcat
2016-02-26 20:06:04
yea
yea
copeland
2016-02-26 20:06:09
21. Let $ABCD$ be a unit square. Let $Q_1$ be the midpoint of $\overline{CD}$. For $i=1,2,\ldots$, let $P_i$ be the intersection of $\overline{AQ_i}$ and $\overline{BD}$, and let $Q_{i+1}$ be the foot of the perpendicular from $P_i$ to $\overline{CD}$. What is \[ \sum_{i=1}^\infty \text{Area of } \triangle DQ_iP_i\;?\]
(A) $\dfrac16$ (B) $\dfrac14$ (C) $\dfrac13$ (D) $\dfrac12$ (E) $1$
21. Let $ABCD$ be a unit square. Let $Q_1$ be the midpoint of $\overline{CD}$. For $i=1,2,\ldots$, let $P_i$ be the intersection of $\overline{AQ_i}$ and $\overline{BD}$, and let $Q_{i+1}$ be the foot of the perpendicular from $P_i$ to $\overline{CD}$. What is \[ \sum_{i=1}^\infty \text{Area of } \triangle DQ_iP_i\;?\]
(A) $\dfrac16$ (B) $\dfrac14$ (C) $\dfrac13$ (D) $\dfrac12$ (E) $1$
copeland
2016-02-26 20:06:34
Maybe this is a geometry problem. There are lots of letters. . .
Maybe this is a geometry problem. There are lots of letters. . .
phi_ftw1618
2016-02-26 20:06:46
diagram?
diagram?
temp8909
2016-02-26 20:06:46
DIAGRAM!!!!
DIAGRAM!!!!
Puzzled417
2016-02-26 20:06:46
draw a picture
draw a picture
greenguy03
2016-02-26 20:06:46
Diagram!
Diagram!
TheRealDeal
2016-02-26 20:06:46
diagram
diagram
yrnsmurf
2016-02-26 20:06:46
diagram!
diagram!
_--__--_
2016-02-26 20:06:46
Diagram
Diagram
copeland
2016-02-26 20:06:49
Oh, for sure.
Oh, for sure.
copeland
2016-02-26 20:06:53
Now it's definitely geometry.
Now it's definitely geometry.
copeland
2016-02-26 20:06:56
Let's draw a picture on the coordinate plane. We'll start with the square. I put $A$ at the origin, mainly because I thought having one endpoint of $\overline{AQ_i}$ always be the origin would be useful, but it probably doesn't matter too much.
Let's draw a picture on the coordinate plane. We'll start with the square. I put $A$ at the origin, mainly because I thought having one endpoint of $\overline{AQ_i}$ always be the origin would be useful, but it probably doesn't matter too much.
copeland
2016-02-26 20:06:57
copeland
2016-02-26 20:07:17
Should I add some Ps and Qs?
Should I add some Ps and Qs?
copeland
2016-02-26 20:07:19
Do you mind?
Do you mind?
Puzzled417
2016-02-26 20:07:23
us coordinate geometry
us coordinate geometry
copeland
2016-02-26 20:07:26
Ooh. coordinates.
Ooh. coordinates.
Aathreyakadambi
2016-02-26 20:07:29
SURE!
SURE!
TheRealDeal
2016-02-26 20:07:29
add them
add them
copeland
2016-02-26 20:07:32
copeland
2016-02-26 20:07:37
What do you notice?
What do you notice?
copeland
2016-02-26 20:08:15
First off, a lot of people are saying "similar triangles."
First off, a lot of people are saying "similar triangles."
copeland
2016-02-26 20:08:18
Are those similar?
Are those similar?
Burrito
2016-02-26 20:08:54
nope
nope
temp8909
2016-02-26 20:08:54
no
no
SineWarrior
2016-02-26 20:08:54
no
no
Tapeman
2016-02-26 20:08:54
no
no
jkoj25
2016-02-26 20:08:54
no
no
Jyzhang12
2016-02-26 20:08:54
no
no
agbdmrbirdyface
2016-02-26 20:08:54
wait no
wait no
jkoj25
2016-02-26 20:08:54
the angles are different
the angles are different
SK200
2016-02-26 20:08:54
no
no
Pisensei
2016-02-26 20:08:54
nope
nope
physangel
2016-02-26 20:08:54
sorry, no
sorry, no
Jayjayliu
2016-02-26 20:08:54
no
no
Rotack00
2016-02-26 20:08:54
no, angles are different?
no, angles are different?
copeland
2016-02-26 20:08:55
No. The angle at Q gets closer and closer to 90 degrees.
No. The angle at Q gets closer and closer to 90 degrees.
copeland
2016-02-26 20:09:03
If we let $q_i$ be the $y$-coordinate of $Q_i$, then what's the area of $\triangle DQ_iP_i$?
If we let $q_i$ be the $y$-coordinate of $Q_i$, then what's the area of $\triangle DQ_iP_i$?
copeland
2016-02-26 20:09:16
It looks like we want base multiplied by height.
It looks like we want base multiplied by height.
copeland
2016-02-26 20:09:22
What base should we use?
What base should we use?
temp8909
2016-02-26 20:09:47
$QD$
$QD$
yrnsmurf
2016-02-26 20:09:47
DQi
DQi
greenguy03
2016-02-26 20:09:47
DQi
DQi
agbdmrbirdyface
2016-02-26 20:09:47
we should use DQi
we should use DQi
copeland
2016-02-26 20:09:54
What's that length?
What's that length?
copeland
2016-02-26 20:10:35
(as a function of i)
(as a function of i)
copeland
2016-02-26 20:10:59
Sorry, that question was harder than I meant it to be.
Sorry, that question was harder than I meant it to be.
copeland
2016-02-26 20:11:14
The distance from the point (1,0) to the point $(1,q_i)$ is. .
The distance from the point (1,0) to the point $(1,q_i)$ is. .
swagger21
2016-02-26 20:11:39
q_i
q_i
temp8909
2016-02-26 20:11:39
$q_i$
$q_i$
agbdmrbirdyface
2016-02-26 20:11:39
qi
qi
Dominater76
2016-02-26 20:11:39
$q_i$
$q_i$
ninjataco
2016-02-26 20:11:39
q_i
q_i
Jayjayliu
2016-02-26 20:11:39
q_i
q_i
greenguy03
2016-02-26 20:11:39
$qi$
$qi$
copeland
2016-02-26 20:11:42
The base of $\triangle DQ_iP_i$ is $DQ_i$, which has length $q_i$.
The base of $\triangle DQ_iP_i$ is $DQ_i$, which has length $q_i$.
copeland
2016-02-26 20:11:49
The height of $\triangle DQ_iP_i$ is $P_iQ_{i+1}$.
The height of $\triangle DQ_iP_i$ is $P_iQ_{i+1}$.
copeland
2016-02-26 20:11:54
That one is harder.
That one is harder.
copeland
2016-02-26 20:12:02
How else can we express the length $P_iQ_{i+1}$?
How else can we express the length $P_iQ_{i+1}$?
copeland
2016-02-26 20:12:05
What do we know about triangle $DP_iQ_{i+1}$?
What do we know about triangle $DP_iQ_{i+1}$?
greenguy03
2016-02-26 20:12:28
It's right
It's right
yrnsmurf
2016-02-26 20:12:28
It's a right triangle
It's a right triangle
swagger21
2016-02-26 20:12:28
it's right
it's right
Rotack00
2016-02-26 20:12:28
its a right triangle?
its a right triangle?
Dominater76
2016-02-26 20:12:28
Right triangle?
Right triangle?
_--__--_
2016-02-26 20:12:28
right triangle
right triangle
copeland
2016-02-26 20:12:30
It's more than just right. . .
It's more than just right. . .
copeland
2016-02-26 20:12:48
It's, like, annoying about how right it is. Like it keeps telling you how wrong you are. . .
It's, like, annoying about how right it is. Like it keeps telling you how wrong you are. . .
TheRealDeal
2016-02-26 20:12:58
right isoscelese triangle
right isoscelese triangle
goldentail141
2016-02-26 20:12:58
45-45-90 right triangle
45-45-90 right triangle
gradysocool
2016-02-26 20:12:58
isoceles right
isoceles right
swagger21
2016-02-26 20:12:58
isoceles right triangle
isoceles right triangle
_--__--_
2016-02-26 20:12:58
isosceles right?
isosceles right?
math101010
2016-02-26 20:12:58
isosceles
isosceles
ghghghghghghghgh
2016-02-26 20:12:58
isosceles right
isosceles right
temp8909
2016-02-26 20:12:58
isosceles right!
isosceles right!
avn
2016-02-26 20:12:58
isoceles
isoceles
copeland
2016-02-26 20:13:08
Uh, I mean, it's an isosceles right triangle! (Since $\angle BDC = 45^\circ$.)
Uh, I mean, it's an isosceles right triangle! (Since $\angle BDC = 45^\circ$.)
copeland
2016-02-26 20:13:14
So $P_iQ_{i+1}$ has the same length as $DQ_{i+1}$. Thus, the height of $\triangle DQ_iP_i$ is $P_iQ_{i+1} = DQ_{i+1} = q_{i+1}$.
So $P_iQ_{i+1}$ has the same length as $DQ_{i+1}$. Thus, the height of $\triangle DQ_iP_i$ is $P_iQ_{i+1} = DQ_{i+1} = q_{i+1}$.
copeland
2016-02-26 20:13:16
So, the area of $\triangle DQ_iP_i$ is $\frac12 q_iq_{i+1}$, and the sum we want is \[ \frac12 \sum_{i=1}^\infty q_iq_{i+1}.\]
So, the area of $\triangle DQ_iP_i$ is $\frac12 q_iq_{i+1}$, and the sum we want is \[ \frac12 \sum_{i=1}^\infty q_iq_{i+1}.\]
copeland
2016-02-26 20:13:25
We know $q_1 = \frac12$. How can we determine the other $q$'s?
We know $q_1 = \frac12$. How can we determine the other $q$'s?
copeland
2016-02-26 20:14:09
At some point we need to do some arithmetic on this test. . .
At some point we need to do some arithmetic on this test. . .
chenmeister22
2016-02-26 20:14:30
Coordinate Geometry
Coordinate Geometry
Puzzled417
2016-02-26 20:14:30
use coordinate geometry?
use coordinate geometry?
thepiercingarrow
2016-02-26 20:14:33
calculator!
calculator!
copeland
2016-02-26 20:14:35
Weak.
Weak.
copeland
2016-02-26 20:14:49
I don't mean "You are weak"
I don't mean "You are weak"
copeland
2016-02-26 20:14:53
I mean, um, something else.
I mean, um, something else.
DashK
2016-02-26 20:15:00
graphic calculator!
graphic calculator!
copeland
2016-02-26 20:15:02
. . .
. . .
copeland
2016-02-26 20:15:07
Who's trolling whom again?
Who's trolling whom again?
copeland
2016-02-26 20:15:19
What line is $AQ_i$?
What line is $AQ_i$?
techguy2
2016-02-26 20:15:31
um
um
copeland
2016-02-26 20:15:57
What is its equation.
What is its equation.
copeland
2016-02-26 20:16:08
I'll give you one point: $A=(0,0)$.
I'll give you one point: $A=(0,0)$.
chenmeister22
2016-02-26 20:16:40
$y=\frac{x}{2}$
$y=\frac{x}{2}$
yrnsmurf
2016-02-26 20:16:40
y=x/2
y=x/2
mathguy5041
2016-02-26 20:16:40
y=1/2x
y=1/2x
copeland
2016-02-26 20:16:46
OK, but we need to do more than just the first one.
OK, but we need to do more than just the first one.
copeland
2016-02-26 20:16:50
What about for general $i$?
What about for general $i$?
SK200
2016-02-26 20:17:02
y=(q_i)(x)
y=(q_i)(x)
agbdmrbirdyface
2016-02-26 20:17:04
$ y = q_i * x
$ y = q_i * x
copeland
2016-02-26 20:17:10
Great.
Great.
copeland
2016-02-26 20:17:13
We know that line $AQ_i$ is just the line $y = q_ix$.
We know that line $AQ_i$ is just the line $y = q_ix$.
copeland
2016-02-26 20:17:21
And what is line $BD?$
And what is line $BD?$
agbdmrbirdyface
2016-02-26 20:17:59
y = -x + 1
y = -x + 1
yrnsmurf
2016-02-26 20:17:59
x+y=1
x+y=1
goldentail141
2016-02-26 20:17:59
$y = 1 - x$
$y = 1 - x$
Jayjayliu
2016-02-26 20:17:59
y=1-x
y=1-x
ghghghghghghghgh
2016-02-26 20:17:59
y=-x+1
y=-x+1
swagger21
2016-02-26 20:17:59
y = -x + 1
y = -x + 1
temp8909
2016-02-26 20:17:59
$y=1-x$
$y=1-x$
ein
2016-02-26 20:17:59
-x+1=y
-x+1=y
mathguy5041
2016-02-26 20:17:59
$x+y=1$
$x+y=1$
greenguy03
2016-02-26 20:17:59
With the equation y = -x+1
With the equation y = -x+1
_--__--_
2016-02-26 20:17:59
$y = -x + 1$
$y = -x + 1$
weeshree
2016-02-26 20:17:59
y=-x+1
y=-x+1
sillyd
2016-02-26 20:18:02
y=-x+1
y=-x+1
copeland
2016-02-26 20:18:04
$\overline{BD}$ is part of the line $x+y = 1$.
$\overline{BD}$ is part of the line $x+y = 1$.
copeland
2016-02-26 20:18:06
So $q_{i+1}$ is $y$-coordinate of the intersection of the lines
\begin{align*}
x+y&=1\\
y&=q_ix.
\end{align*}
So $q_{i+1}$ is $y$-coordinate of the intersection of the lines
\begin{align*}
x+y&=1\\
y&=q_ix.
\end{align*}
copeland
2016-02-26 20:18:10
We want to eliminate $x$ so we multiply the first by $q_i$ and substitute.
We want to eliminate $x$ so we multiply the first by $q_i$ and substitute.
copeland
2016-02-26 20:18:17
So $y=q_{i+1}$ solves \[q_ix+q_iy=q_i.\] Substituting $y$ for $q_ix$ and setting $y=q_{i+1}$ gives us the recurrence\[q_{i+1}+q_iq_{i+1}=q_{i}.\]
So $y=q_{i+1}$ solves \[q_ix+q_iy=q_i.\] Substituting $y$ for $q_ix$ and setting $y=q_{i+1}$ gives us the recurrence\[q_{i+1}+q_iq_{i+1}=q_{i}.\]
copeland
2016-02-26 20:18:42
Now we could solve this for $q_{i+1}$, but what's a more clever move?
Now we could solve this for $q_{i+1}$, but what's a more clever move?
copeland
2016-02-26 20:19:08
What are we trying to find?
What are we trying to find?
mathwizard888
2016-02-26 20:19:57
$q_iq_{i+1}=q_i-q_{i+1}$ and we can telescope
$q_iq_{i+1}=q_i-q_{i+1}$ and we can telescope
goldentail141
2016-02-26 20:19:57
Write the equation as $q_{i}q_{i + 1} = q_{1} - q_{i + 1}.$
Write the equation as $q_{i}q_{i + 1} = q_{1} - q_{i + 1}.$
mathguy5041
2016-02-26 20:19:57
subtract q_i+1 from both sides
subtract q_i+1 from both sides
temp8909
2016-02-26 20:19:57
$q_iq_{i+1}$
$q_iq_{i+1}$
legozelda
2016-02-26 20:19:57
find q(i)*q(i+1)
find q(i)*q(i+1)
agbdmrbirdyface
2016-02-26 20:19:57
oh wait qi * qi+1
oh wait qi * qi+1
yrnsmurf
2016-02-26 20:19:57
subtract $(q_i+1)$ from each side
subtract $(q_i+1)$ from each side
ein
2016-02-26 20:19:57
Substitute q_iq_(i+1) into the sum
Substitute q_iq_(i+1) into the sum
mathguy5041
2016-02-26 20:20:06
telescoping sum
telescoping sum
agbdmrbirdyface
2016-02-26 20:20:06
ohhhh we subtract q(i+1) from both sides
ohhhh we subtract q(i+1) from both sides
idomath12345
2016-02-26 20:20:06
Then cancel like mad.
Then cancel like mad.
copeland
2016-02-26 20:20:08
Our recurrence simplifies to $q_iq_{i+1} = q_i - q_{i+1}$.
Our recurrence simplifies to $q_iq_{i+1} = q_i - q_{i+1}$.
copeland
2016-02-26 20:20:10
So our sum is a telescoping series! \[ \frac12 \sum_{i=1}^\infty (q_i - q_{i+1}).\]
So our sum is a telescoping series! \[ \frac12 \sum_{i=1}^\infty (q_i - q_{i+1}).\]
copeland
2016-02-26 20:20:11
We're home free now. What does it sum to?
We're home free now. What does it sum to?
techguy2
2016-02-26 20:20:33
Can you like, sticky the question? that would help.
Can you like, sticky the question? that would help.
techguy2
2016-02-26 20:20:45
so we don't have to scroll.
so we don't have to scroll.
W.Sun
2016-02-26 20:20:53
1/4
1/4
Jayjayliu
2016-02-26 20:20:53
1/4
1/4
mathguy5041
2016-02-26 20:20:53
1/4
1/4
Readingrocks88
2016-02-26 20:20:53
1/4
1/4
W.Sun
2016-02-26 20:20:53
Telescoping!!!! 1/4!! :spidey:
Telescoping!!!! 1/4!! :spidey:
idomath12345
2016-02-26 20:20:53
JUST 1/2*q1?
JUST 1/2*q1?
goldentail141
2016-02-26 20:20:53
$\frac{1}{4}$
$\frac{1}{4}$
gradysocool
2016-02-26 20:20:53
1/2 qi=1/4
1/2 qi=1/4
temp8909
2016-02-26 20:20:53
$\frac{1}{2}q_1=\frac{1}{4}$
$\frac{1}{2}q_1=\frac{1}{4}$
yrnsmurf
2016-02-26 20:20:53
$\dfrac14$
$\dfrac14$
copeland
2016-02-26 20:20:55
The answer is just $\dfrac12 q_1 = \boxed{\dfrac14}$. Answer (B).
The answer is just $\dfrac12 q_1 = \boxed{\dfrac14}$. Answer (B).
copeland
2016-02-26 20:21:09
Alright, any questions?
Alright, any questions?
mathguy5041
2016-02-26 20:21:35
On to 22!
On to 22!
gradysocool
2016-02-26 20:22:10
Is there a purely geometric way of doing the last one?
Is there a purely geometric way of doing the last one?
copeland
2016-02-26 20:22:14
That's a fantastic question.
That's a fantastic question.
copeland
2016-02-26 20:22:16
I'd love to see one.
I'd love to see one.
copeland
2016-02-26 20:22:26
I wouldn't be too surprised, now that you mention it.
I wouldn't be too surprised, now that you mention it.
copeland
2016-02-26 20:22:52
I'd probably start by thinking about the respective areas and then try to find equal areas that fill, maybe, the big red triangle.
I'd probably start by thinking about the respective areas and then try to find equal areas that fill, maybe, the big red triangle.
copeland
2016-02-26 20:23:15
Like, maybe the red triangles are the right areas and then you're done? Give it a whirl. . .
Like, maybe the red triangles are the right areas and then you're done? Give it a whirl. . .
copeland
2016-02-26 20:23:25
Oh, that is true.
Oh, that is true.
copeland
2016-02-26 20:23:56
$(q_i-q_{i+1})/2$ is the area of one of those triangles. Neat.
$(q_i-q_{i+1})/2$ is the area of one of those triangles. Neat.
copeland
2016-02-26 20:24:00
I'd love to see the rest.
I'd love to see the rest.
steveshaff
2016-02-26 20:24:08
There is...check out my solution on the contest forum
There is...check out my solution on the contest forum
copeland
2016-02-26 20:24:15
Apparently it exists out there in the interwebs.
Apparently it exists out there in the interwebs.
copeland
2016-02-26 20:24:21
Problem 22?
Problem 22?
copeland
2016-02-26 20:24:23
22. For a certain positive integer $n$ less than 1000, the decimal equivalent of $\frac{1}{n}$ is $0.\overline{abcdef}$, a repeating decimal of period 6, and the decimal equivalent of $\frac{1}{n+6}$ is $0.\overline{wxyz}$, a repeating decimal of period 4. In which interval does $n$ lie?
$\phantom{12B:22}$
(A) [1,200] (B) [201,400] (C) [401,600] (D) [601,800] (E) [801,999]
22. For a certain positive integer $n$ less than 1000, the decimal equivalent of $\frac{1}{n}$ is $0.\overline{abcdef}$, a repeating decimal of period 6, and the decimal equivalent of $\frac{1}{n+6}$ is $0.\overline{wxyz}$, a repeating decimal of period 4. In which interval does $n$ lie?
$\phantom{12B:22}$
(A) [1,200] (B) [201,400] (C) [401,600] (D) [601,800] (E) [801,999]
copeland
2016-02-26 20:24:24
How can we work with repeating decimals?
How can we work with repeating decimals?
Dominater76
2016-02-26 20:25:59
With lots of 9s
With lots of 9s
ptes77
2016-02-26 20:25:59
Turn into fraction form
Turn into fraction form
weeshree
2016-02-26 20:25:59
express as fractions
express as fractions
yrnsmurf
2016-02-26 20:25:59
it is equal to $\dfrac{\overline{abcdef}}{999999}$
it is equal to $\dfrac{\overline{abcdef}}{999999}$
jkoj25
2016-02-26 20:25:59
999999
999999
copeland
2016-02-26 20:26:09
A repeating decimal can be written as a fraction with a bunch of 9's in the denominator. For example, $0.22222\ldots = \dfrac29$ and $0.137137137\ldots = \dfrac{137}{999}$.
A repeating decimal can be written as a fraction with a bunch of 9's in the denominator. For example, $0.22222\ldots = \dfrac29$ and $0.137137137\ldots = \dfrac{137}{999}$.
copeland
2016-02-26 20:26:15
So we can write $\dfrac{1}{n} =\dfrac{r}{999999}$ and $\dfrac{1}{n+6} = \dfrac{s}{9999}$, where $r$ is the number $abcdef$ and $s$ is the number $wxyz$.
So we can write $\dfrac{1}{n} =\dfrac{r}{999999}$ and $\dfrac{1}{n+6} = \dfrac{s}{9999}$, where $r$ is the number $abcdef$ and $s$ is the number $wxyz$.
copeland
2016-02-26 20:26:17
How does that help?
How does that help?
MSTang
2016-02-26 20:27:25
$n$ divides 999999 and $n+6$ divides 9999
$n$ divides 999999 and $n+6$ divides 9999
temp8909
2016-02-26 20:27:25
n is a factor of 999999 and n+6 is a factor of 9999
n is a factor of 999999 and n+6 is a factor of 9999
macandcheese
2016-02-26 20:27:25
n must divide 999999, n+6 must divide 9999
n must divide 999999, n+6 must divide 9999
SK200
2016-02-26 20:27:25
$rn=999999$&$s(n+6)=9999$
$rn=999999$&$s(n+6)=9999$
summitwei
2016-02-26 20:27:25
n is a factor of 999999 and n+6 is a factor of 9999
n is a factor of 999999 and n+6 is a factor of 9999
yrnsmurf
2016-02-26 20:27:25
n is divisor of 999999 while n+6 is divisor of 9999
n is divisor of 999999 while n+6 is divisor of 9999
copeland
2016-02-26 20:27:27
These give $999999 = rn$ and $9999 = s(n+6)$. In particular, $n$ is a factor of 999999 and $n+6$ is a factor of 9999.
These give $999999 = rn$ and $9999 = s(n+6)$. In particular, $n$ is a factor of 999999 and $n+6$ is a factor of 9999.
copeland
2016-02-26 20:27:29
Which of these is likely to be easier to work with?
Which of these is likely to be easier to work with?
mathguy5041
2016-02-26 20:28:38
9999
9999
greenguy03
2016-02-26 20:28:38
The n+6
The n+6
summitwei
2016-02-26 20:28:38
9999 because it's smaller
9999 because it's smaller
SherlockHolmes7
2016-02-26 20:28:38
9999
9999
sillyd
2016-02-26 20:28:38
9999
9999
mathguy5041
2016-02-26 20:28:38
since it has less factors, 9999
since it has less factors, 9999
yrnsmurf
2016-02-26 20:28:38
9999 because 101 and 11
9999 because 101 and 11
illogical_21
2016-02-26 20:28:39
9999=s(n+6)
9999=s(n+6)
copeland
2016-02-26 20:28:41
9999 is smaller, so it probably has fewer factors.
9999 is smaller, so it probably has fewer factors.
copeland
2016-02-26 20:28:42
Indeed, $9999 = 99 \cdot 101 = 9 \cdot 11 \cdot 101$.
Indeed, $9999 = 99 \cdot 101 = 9 \cdot 11 \cdot 101$.
copeland
2016-02-26 20:28:44
However, what else do we know about $n+6$?
However, what else do we know about $n+6$?
copeland
2016-02-26 20:28:50
Can it be any of those factors?
Can it be any of those factors?
thkim1011
2016-02-26 20:30:15
therefore n+6 is one of 101, 303, 909
therefore n+6 is one of 101, 303, 909
thkim1011
2016-02-26 20:30:15
it must contain a 101 else the period is not 4
it must contain a 101 else the period is not 4
temp8909
2016-02-26 20:30:15
it can't divide 9 or 99
it can't divide 9 or 99
MSTang
2016-02-26 20:30:15
99 = 9 * 11, so n+6 can't also divide 99 (4 is the minimal period)
99 = 9 * 11, so n+6 can't also divide 99 (4 is the minimal period)
copeland
2016-02-26 20:30:22
It can't be a factor of $99$.
It can't be a factor of $99$.
copeland
2016-02-26 20:30:23
If it were, then we could write $\dfrac{1}{n+6} = \dfrac{t}{99}$ for some $t$, and in particular the decimal $\dfrac{1}{n+6}$ would only have period 2 (or 1).
If it were, then we could write $\dfrac{1}{n+6} = \dfrac{t}{99}$ for some $t$, and in particular the decimal $\dfrac{1}{n+6}$ would only have period 2 (or 1).
copeland
2016-02-26 20:30:25
So we must have $n+6 = 101k$ for some $k$ that divides 99.
So we must have $n+6 = 101k$ for some $k$ that divides 99.
copeland
2016-02-26 20:30:27
Since we're looking for $n < 1000$, that only leaves $n+6 \in \{101, 303, 909\}$ as possibilities.
Since we're looking for $n < 1000$, that only leaves $n+6 \in \{101, 303, 909\}$ as possibilities.
copeland
2016-02-26 20:30:28
That is, $n \in \{95, 297, 903\}$.
That is, $n \in \{95, 297, 903\}$.
copeland
2016-02-26 20:30:30
How can we tell which one works?
How can we tell which one works?
temp8909
2016-02-26 20:31:21
divide them into 999999
divide them into 999999
Couper
2016-02-26 20:31:21
3 possibilities so just check the cases
3 possibilities so just check the cases
MSTang
2016-02-26 20:31:21
try dividing into 999999
try dividing into 999999
agbdmrbirdyface
2016-02-26 20:31:21
we have to go over to the other equation?
we have to go over to the other equation?
macandcheese
2016-02-26 20:31:21
297 divides 999999
297 divides 999999
thkim1011
2016-02-26 20:31:25
297 = 27 * 11 so has period 3 * 2
297 = 27 * 11 so has period 3 * 2
copeland
2016-02-26 20:31:33
We need $n$ to be a factor of 999999. So we can check them all.
We need $n$ to be a factor of 999999. So we can check them all.
copeland
2016-02-26 20:31:34
Clearly 95 is not a factor of 999999, since 5 doesn't divide 999999.
Clearly 95 is not a factor of 999999, since 5 doesn't divide 999999.
copeland
2016-02-26 20:31:35
297 works! $297 = 3 \cdot 99$, and thus $999999 \div 297 = 10101 \div 3$ is an integer.
297 works! $297 = 3 \cdot 99$, and thus $999999 \div 297 = 10101 \div 3$ is an integer.
copeland
2016-02-26 20:31:38
Finally, $903 = 3 \cdot 301 = 3 \cdot 7 \cdot 43$ doesn't work, since 43 is not a factor of 999999.
Finally, $903 = 3 \cdot 301 = 3 \cdot 7 \cdot 43$ doesn't work, since 43 is not a factor of 999999.
copeland
2016-02-26 20:31:44
So $n = 297$ must be the number we're looking for. This is in the interval $\boxed{[201,400]}$. Answer (B).
So $n = 297$ must be the number we're looking for. This is in the interval $\boxed{[201,400]}$. Answer (B).
mathguy5041
2016-02-26 20:32:20
As a check the abcdef is 003367
As a check the abcdef is 003367
mathguy5041
2016-02-26 20:32:20
ah, 3367, the magical number
ah, 3367, the magical number
copeland
2016-02-26 20:32:31
Note that we never verified that $\dfrac{1}{297}$ has period 6 -- we only learned that its period is at most 6. If 297 were a factor of a smaller number of 9's, then the period would be smaller.
Note that we never verified that $\dfrac{1}{297}$ has period 6 -- we only learned that its period is at most 6. If 297 were a factor of a smaller number of 9's, then the period would be smaller.
copeland
2016-02-26 20:32:33
But since we checked that the other possible values of $n$ definitely didn't work, $n=297$ must be the answer. Just ask Sherlock Holmes: "Once you eliminate the impossible, whatever remains, no matter how improbable, must be the truth."
But since we checked that the other possible values of $n$ definitely didn't work, $n=297$ must be the answer. Just ask Sherlock Holmes: "Once you eliminate the impossible, whatever remains, no matter how improbable, must be the truth."
copeland
2016-02-26 20:32:39
23. What is the volume of the region in three-dimensional space defined by the inequalities $|x| + |y| + |z| \le 1$ and $|x| + |y| + |z-1| \le 1$?
$\phantom{12B:23}$
(A) $\dfrac16$ (B) $\dfrac13$ (C) $\dfrac12$ (D) $\dfrac23$ (E) $1$
23. What is the volume of the region in three-dimensional space defined by the inequalities $|x| + |y| + |z| \le 1$ and $|x| + |y| + |z-1| \le 1$?
$\phantom{12B:23}$
(A) $\dfrac16$ (B) $\dfrac13$ (C) $\dfrac12$ (D) $\dfrac23$ (E) $1$
copeland
2016-02-26 20:32:52
This problem is vaguely reminiscent of 12B #18 / 10B #21.
This problem is vaguely reminiscent of 12B #18 / 10B #21.
copeland
2016-02-26 20:33:50
Can we use a similar trick?
Can we use a similar trick?
idomath12345
2016-02-26 20:34:22
Yes?
Yes?
copeland
2016-02-26 20:34:25
I hope so. . .
I hope so. . .
copeland
2016-02-26 20:34:29
We had some symmetry before.
We had some symmetry before.
copeland
2016-02-26 20:34:33
Do we have any symmetry now?
Do we have any symmetry now?
agbdmrbirdyface
2016-02-26 20:35:26
kind of, in the first equation
kind of, in the first equation
agbdmrbirdyface
2016-02-26 20:35:26
but not in the second one
but not in the second one
avn
2016-02-26 20:35:26
yes
yes
SineWarrior
2016-02-26 20:35:26
yes and no
yes and no
copeland
2016-02-26 20:35:43
Let's just look at $x$. Is there any symmetry (in BOTH equations) with respect to $x$?
Let's just look at $x$. Is there any symmetry (in BOTH equations) with respect to $x$?
SherlockHolmes7
2016-02-26 20:36:48
yes
yes
temp8909
2016-02-26 20:36:48
yes!!!
yes!!!
swagger21
2016-02-26 20:36:48
yes?
yes?
copeland
2016-02-26 20:36:54
So what does that tells us?
So what does that tells us?
summitwei
2016-02-26 20:37:35
We only need to work with x>0 and then multiply by 2
We only need to work with x>0 and then multiply by 2
copeland
2016-02-26 20:37:40
This is symmetric with respect to $x\mapsto -x$, so we can just restrict to $x>0$ and multiply by 2 at the end.
This is symmetric with respect to $x\mapsto -x$, so we can just restrict to $x>0$ and multiply by 2 at the end.
copeland
2016-02-26 20:37:44
Right?
Right?
copeland
2016-02-26 20:37:47
What else?
What else?
Couper
2016-02-26 20:38:19
Symmetry with respect to y
Symmetry with respect to y
idomath12345
2016-02-26 20:38:19
y is same.
y is same.
swagger21
2016-02-26 20:38:19
it is symmetric with respect to y?
it is symmetric with respect to y?
yrnsmurf
2016-02-26 20:38:19
Same thing with y
Same thing with y
macandcheese
2016-02-26 20:38:19
same with y
same with y
idomath12345
2016-02-26 20:38:19
y has symmetry.
y has symmetry.
legozelda
2016-02-26 20:38:19
same for y>0, then multiply by 2(multiply by 4 total)
same for y>0, then multiply by 2(multiply by 4 total)
weeshree
2016-02-26 20:38:19
restrict y similarly
restrict y similarly
gradysocool
2016-02-26 20:38:19
The y's as well
The y's as well
copeland
2016-02-26 20:38:21
This is symmetric with respect to $y\mapsto -y$, so we can just restrict to $y>0$ and multiply by 2 at the end also!
This is symmetric with respect to $y\mapsto -y$, so we can just restrict to $y>0$ and multiply by 2 at the end also!
copeland
2016-02-26 20:38:27
Great. And also $z?$
Great. And also $z?$
legozelda
2016-02-26 20:38:54
no
no
temp8909
2016-02-26 20:38:54
not so much!
not so much!
legozelda
2016-02-26 20:38:54
kind of?
kind of?
agbdmrbirdyface
2016-02-26 20:38:54
no
no
W.Sun
2016-02-26 20:38:54
Not with Z
Not with Z
chenmeister22
2016-02-26 20:38:54
Nope
Nope
copeland
2016-02-26 20:39:01
We can just compute the region where $x$ and $y$ are positive, and multiply by 4.
We can just compute the region where $x$ and $y$ are positive, and multiply by 4.
copeland
2016-02-26 20:39:01
But we don't have that flexibility with $z$.
But we don't have that flexibility with $z$.
copeland
2016-02-26 20:39:03
Or do we? Is there a symmetric with $z$ that we can exploit?
Or do we? Is there a symmetric with $z$ that we can exploit?
copeland
2016-02-26 20:39:06
What can we replace $z$ with and not change the inequalities?
What can we replace $z$ with and not change the inequalities?
agbdmrbirdyface
2016-02-26 20:39:53
(1-z)?
(1-z)?
temp8909
2016-02-26 20:39:53
reflection via $z=\frac{1}{2}$
reflection via $z=\frac{1}{2}$
mssmath
2016-02-26 20:39:53
1-z
1-z
yrnsmurf
2016-02-26 20:39:53
1-z
1-z
copeland
2016-02-26 20:39:59
We can replace $z$ with $1-z$, and vice versa. Note that $|1-z| = |z-1|$ and $|(1-z)-1| = |-z| = |z|$, so replacing $z$ with $1-z$ just interchanges the two inequalities.
We can replace $z$ with $1-z$, and vice versa. Note that $|1-z| = |z-1|$ and $|(1-z)-1| = |-z| = |z|$, so replacing $z$ with $1-z$ just interchanges the two inequalities.
copeland
2016-02-26 20:39:59
Thus there's a third symmetry.
Thus there's a third symmetry.
weeshree
2016-02-26 20:40:12
z>1/2
z>1/2
illogical_21
2016-02-26 20:40:12
z-1/2
z-1/2
Couper
2016-02-26 20:40:12
z -> z-1/2
z -> z-1/2
weeshree
2016-02-26 20:40:12
q = z-1/2
q = z-1/2
copeland
2016-02-26 20:40:16
Note that $z = 1-z$ is true along $z= \dfrac12$, so replacing $z$ with $1-z$ is the same as reflecting across the $z = \dfrac12$ plane.
Note that $z = 1-z$ is true along $z= \dfrac12$, so replacing $z$ with $1-z$ is the same as reflecting across the $z = \dfrac12$ plane.
copeland
2016-02-26 20:40:24
Thus, our solid consists of 8 identical pieces, symmetric across the $x=0$, $y=0$, and $z=\dfrac12$ planes.
Thus, our solid consists of 8 identical pieces, symmetric across the $x=0$, $y=0$, and $z=\dfrac12$ planes.
copeland
2016-02-26 20:40:24
We can just compute the volume of one of them, and then multiply by 8 to get our final answer.
We can just compute the volume of one of them, and then multiply by 8 to get our final answer.
copeland
2016-02-26 20:40:26
So let's compute the volume of the octant in which $x \ge 0$, $y \ge 0$, and $z \ge \dfrac12$.
So let's compute the volume of the octant in which $x \ge 0$, $y \ge 0$, and $z \ge \dfrac12$.
copeland
2016-02-26 20:40:31
We want the region in which $x + y + z \le 1$ in this octant. What does this region look like?
We want the region in which $x + y + z \le 1$ in this octant. What does this region look like?
LetThePlatPlay
2016-02-26 20:41:30
a tetrahedron
a tetrahedron
legozelda
2016-02-26 20:41:30
triangular pyramid
triangular pyramid
agbdmrbirdyface
2016-02-26 20:41:30
it has to be a pyramid?
it has to be a pyramid?
ein
2016-02-26 20:41:30
a triangular prism
a triangular prism
yrnsmurf
2016-02-26 20:41:30
a pyramid
a pyramid
adas1
2016-02-26 20:41:30
a tetrahedron?
a tetrahedron?
greenguy03
2016-02-26 20:41:30
A triangular pyramid?
A triangular pyramid?
copeland
2016-02-26 20:41:33
In the $z = \dfrac12$ plane, it's the region $x+y \le \dfrac12$. That's a triangle in the $x,y \ge 0$ quadrant.
In the $z = \dfrac12$ plane, it's the region $x+y \le \dfrac12$. That's a triangle in the $x,y \ge 0$ quadrant.
copeland
2016-02-26 20:41:37
And the plane intersects the $z$-axis at $(0,0,1)$.
And the plane intersects the $z$-axis at $(0,0,1)$.
copeland
2016-02-26 20:41:38
So we have the following picture:
So we have the following picture:
copeland
2016-02-26 20:45:07
copeland
2016-02-26 20:45:40
What is the volume?
What is the volume?
illogical_21
2016-02-26 20:46:34
1/48
1/48
mathguy5041
2016-02-26 20:46:34
1/48
1/48
yrnsmurf
2016-02-26 20:46:34
It's still $\dfrac{1}{48}$
It's still $\dfrac{1}{48}$
temp8909
2016-02-26 20:46:34
$\frac{1}{48}$
$\frac{1}{48}$
SherlockHolmes7
2016-02-26 20:46:34
1/48
1/48
copeland
2016-02-26 20:46:45
Its base has area $\dfrac18$ and its height is $\dfrac12$.
Its base has area $\dfrac18$ and its height is $\dfrac12$.
copeland
2016-02-26 20:46:46
Then, its volume is $$\frac13bh = \frac13 \cdot \frac 18 \cdot \frac12 = \frac{1}{48}.$$
Then, its volume is $$\frac13bh = \frac13 \cdot \frac 18 \cdot \frac12 = \frac{1}{48}.$$
copeland
2016-02-26 20:46:47
And to finish?
And to finish?
Couper
2016-02-26 20:47:14
Multiply by 8 to get 1/6!
Multiply by 8 to get 1/6!
sillyd
2016-02-26 20:47:14
multiply by 8 to get 1/6
multiply by 8 to get 1/6
illogical_21
2016-02-26 20:47:14
1/48*8=1/6
1/48*8=1/6
agbdmrbirdyface
2016-02-26 20:47:14
multiply by 8, since 8 fold symmetry
multiply by 8, since 8 fold symmetry
Dominater76
2016-02-26 20:47:14
Multiply by 8
Multiply by 8
baseballcat
2016-02-26 20:47:14
multiply by 8
multiply by 8
mathguy5041
2016-02-26 20:47:14
Times 8?
Times 8?
yrnsmurf
2016-02-26 20:47:14
multiply that by 8
multiply that by 8
ninjataco
2016-02-26 20:47:14
we have 8 of these
we have 8 of these
agbdmrbirdyface
2016-02-26 20:47:14
so that yields 1/6
so that yields 1/6
temp8909
2016-02-26 20:47:14
$\frac{1}{6}$
$\frac{1}{6}$
copeland
2016-02-26 20:47:16
We have eight of these things, so the total volume is $8 \cdot \dfrac{1}{48} = \boxed{\dfrac16}$. Answer (A).
We have eight of these things, so the total volume is $8 \cdot \dfrac{1}{48} = \boxed{\dfrac16}$. Answer (A).
copeland
2016-02-26 20:47:17
You might also have noticed that these 8 octants fit together to form an octahedron, with center $(0,0,\frac12)$ and vertices $(0,0,0)$, $(0,0,1)$, and $\left(\pm\frac12,\pm\frac12,\frac12\right)$. Here's how our first octant fits into it:
You might also have noticed that these 8 octants fit together to form an octahedron, with center $(0,0,\frac12)$ and vertices $(0,0,0)$, $(0,0,1)$, and $\left(\pm\frac12,\pm\frac12,\frac12\right)$. Here's how our first octant fits into it:
copeland
2016-02-26 20:47:21
copeland
2016-02-26 20:47:34
And I bet the perspective is wrong, too.
And I bet the perspective is wrong, too.
copeland
2016-02-26 20:47:50
Oh, I also drew you a picture for that conversation on problem 21, was it?
Oh, I also drew you a picture for that conversation on problem 21, was it?
copeland
2016-02-26 20:47:57
We conjecture that this region:
We conjecture that this region:
copeland
2016-02-26 20:47:58
copeland
2016-02-26 20:48:07
Has the same area as this region:
Has the same area as this region:
copeland
2016-02-26 20:49:47
copeland
2016-02-26 20:50:27
I think that's a really good hint to get us there.
I think that's a really good hint to get us there.
copeland
2016-02-26 20:50:41
All we need to do now is shear the bottom half of the triangle from D over to A.
All we need to do now is shear the bottom half of the triangle from D over to A.
copeland
2016-02-26 20:50:46
Now let's rock number 24:
Now let's rock number 24:
copeland
2016-02-26 20:51:03
24. There are exactly 77,000 ordered quadruples $(a,b,c,d)$ such that $\gcd(a,b,c,d) = 77$ and $\text{lcm}(a,b,c,d) = n$ .What is the smallest possible value of $n$?
$\phantom{12B:24}$
(A) 13,860 (B) 20,790 (C) 21,560 (D) 27,720 (E) 41,580
24. There are exactly 77,000 ordered quadruples $(a,b,c,d)$ such that $\gcd(a,b,c,d) = 77$ and $\text{lcm}(a,b,c,d) = n$ .What is the smallest possible value of $n$?
$\phantom{12B:24}$
(A) 13,860 (B) 20,790 (C) 21,560 (D) 27,720 (E) 41,580
copeland
2016-02-26 20:51:17
What can we do write at the start to simplify the problem?
What can we do write at the start to simplify the problem?
copeland
2016-02-26 20:51:49
wow.
wow.
copeland
2016-02-26 20:51:55
What can we do right at the start to simplify the problem?
What can we do right at the start to simplify the problem?
copeland
2016-02-26 20:52:14
Would you bye. . . text-to-speech fail?
Would you bye. . . text-to-speech fail?
copeland
2016-02-26 20:52:40
Sorry, homophone humor. I'm dun.
Sorry, homophone humor. I'm dun.
legozelda
2016-02-26 20:52:49
use the fact that lcm * gcd = product: 77 * n = abcd
use the fact that lcm * gcd = product: 77 * n = abcd
LetThePlatPlay
2016-02-26 20:52:49
a,b,c,d all divisible by 77
a,b,c,d all divisible by 77
adas1
2016-02-26 20:52:49
abcd = 77n
abcd = 77n
Couper
2016-02-26 20:52:49
Copeland... get it together man
Copeland... get it together man
Tapeman
2016-02-26 20:52:55
77v=a 77x=b 77y=c 77z=d
77v=a 77x=b 77y=c 77z=d
yrnsmurf
2016-02-26 20:52:55
Write a,b,c,d as 77w,77x,77y,77z
Write a,b,c,d as 77w,77x,77y,77z
copeland
2016-02-26 20:52:57
$a$, $b$, $c$, and $d$ all have a common factor of 77. So we can divide any such quadruple by 77.
$a$, $b$, $c$, and $d$ all have a common factor of 77. So we can divide any such quadruple by 77.
copeland
2016-02-26 20:53:13
That divdes the lcm by 77 as well.
That divdes the lcm by 77 as well.
copeland
2016-02-26 20:53:14
So we can instead solve the problem:
So we can instead solve the problem:
copeland
2016-02-26 20:53:15
24X. There are exactly 77,000 ordered quadruples $(a,b,c,d)$ such that $\gcd(a,b,c,d) = 1$ and $\text{lcm}(a,b,c,d) = m$. What is the smallest possible value of $m$?
$\phantom{12B:24X}$
(A) $\dfrac{13,860}{77}$ (B) $\dfrac{20,790}{77}$ (C) $\dfrac{21,560}{77}$ (D) $\dfrac{27,720}{77}$ (E) $\dfrac{41,580}{77}$
24X. There are exactly 77,000 ordered quadruples $(a,b,c,d)$ such that $\gcd(a,b,c,d) = 1$ and $\text{lcm}(a,b,c,d) = m$. What is the smallest possible value of $m$?
$\phantom{12B:24X}$
(A) $\dfrac{13,860}{77}$ (B) $\dfrac{20,790}{77}$ (C) $\dfrac{21,560}{77}$ (D) $\dfrac{27,720}{77}$ (E) $\dfrac{41,580}{77}$
copeland
2016-02-26 20:53:31
There's no real point in dividing those answer choices out -- once we find $m$, it's easier to compute $77m$ to get the original answer choice.
There's no real point in dividing those answer choices out -- once we find $m$, it's easier to compute $77m$ to get the original answer choice.
copeland
2016-02-26 20:53:41
However, a couple of people here at AoPS HQ did this to get a quick feel for the problem. The values are:
However, a couple of people here at AoPS HQ did this to get a quick feel for the problem. The values are:
copeland
2016-02-26 20:53:43
(A) $\dfrac{13,860}{77}$ (B) $\dfrac{20,790}{77}$ (C) $\dfrac{21,560}{77}$ (D) $\dfrac{27,720}{77}$ (E) $\dfrac{41,580}{77}$
(A) $\dfrac{13,860}{77}$ (B) $\dfrac{20,790}{77}$ (C) $\dfrac{21,560}{77}$ (D) $\dfrac{27,720}{77}$ (E) $\dfrac{41,580}{77}$
copeland
2016-02-26 20:53:44
(A) $2^2 \cdot 3^2 \cdot 5$ (B) $2\cdot 3^3 \cdot 5$ (C) $2^3 \cdot 5 \cdot 7$ (D) $2^3 \cdot 3^2 \cdot 5$ (E) $2^2 \cdot 3^3 \cdot 5$
(A) $2^2 \cdot 3^2 \cdot 5$ (B) $2\cdot 3^3 \cdot 5$ (C) $2^3 \cdot 5 \cdot 7$ (D) $2^3 \cdot 3^2 \cdot 5$ (E) $2^2 \cdot 3^3 \cdot 5$
copeland
2016-02-26 20:53:54
Do you see any choices that are just bad?
Do you see any choices that are just bad?
MSTang
2016-02-26 20:55:14
B, C, and E
B, C, and E
sillyd
2016-02-26 20:55:14
C and E are bad
C and E are bad
agbdmrbirdyface
2016-02-26 20:55:14
actually no, scratch that, B and E
actually no, scratch that, B and E
summitwei
2016-02-26 20:55:14
B, 2^3*3*5 is better, C, 2^3*3*5 is better, and E, 2^3*3^2*5 is better
B, 2^3*3*5 is better, C, 2^3*3*5 is better, and E, 2^3*3^2*5 is better
googol.plex
2016-02-26 20:55:14
B, C, and E
B, C, and E
SineWarrior
2016-02-26 20:55:14
C
C
SineWarrior
2016-02-26 20:55:14
c
c
agbdmrbirdyface
2016-02-26 20:55:14
B and E are pretty big
B and E are pretty big
copeland
2016-02-26 20:55:16
Choice B is bad: $2\cdot3^3\cdot5$ is bigger than $2^3\cdot3\cdot5$ and they both have the same size solution set.
Choice B is bad: $2\cdot3^3\cdot5$ is bigger than $2^3\cdot3\cdot5$ and they both have the same size solution set.
copeland
2016-02-26 20:55:28
Choice E is similarly bad.
Choice E is similarly bad.
copeland
2016-02-26 20:55:37
Choice C fails because we skipped 3.
Choice C fails because we skipped 3.
copeland
2016-02-26 20:55:41
Therefore the answer must be A or D.
Therefore the answer must be A or D.
copeland
2016-02-26 20:55:46
Intriguing, AMC dudes.
Intriguing, AMC dudes.
copeland
2016-02-26 20:55:51
Let's keep rocking.
Let's keep rocking.
copeland
2016-02-26 20:55:58
Given some number $m$, how do we count quadruples that have no common factors and lcm $m$?
Given some number $m$, how do we count quadruples that have no common factors and lcm $m$?
MSTang
2016-02-26 20:56:47
Prime factorize $m$
Prime factorize $m$
illogical_21
2016-02-26 20:56:47
factor m ?
factor m ?
SimonSun
2016-02-26 20:56:50
calculate factors
calculate factors
copeland
2016-02-26 20:56:52
Let's look at it one prime at a time.
Let's look at it one prime at a time.
copeland
2016-02-26 20:56:53
Suppose that $m$ has a $p^k$ term in its prime factorization. In how many different ways can powers of $p$ appear in the prime factorizations of $a,b,c,d$?
Suppose that $m$ has a $p^k$ term in its prime factorization. In how many different ways can powers of $p$ appear in the prime factorizations of $a,b,c,d$?
copeland
2016-02-26 20:56:59
To put it another way, suppose:
$a$ has a $p^w$ term
$b$ has a $p^x$ term
$c$ has a $p^y$ term
$d$ has a $p^z$ term
What are the conditions on $(w,x,y,z)$ so that the gcd is 1 and that the lcm has a $p^k$ term?
To put it another way, suppose:
$a$ has a $p^w$ term
$b$ has a $p^x$ term
$c$ has a $p^y$ term
$d$ has a $p^z$ term
What are the conditions on $(w,x,y,z)$ so that the gcd is 1 and that the lcm has a $p^k$ term?
pinkrock
2016-02-26 20:58:22
w, x, y, or z is 0
w, x, y, or z is 0
summitwei
2016-02-26 20:58:22
min(w,x,y,z)=0 and max(w,x,y,z)=k
min(w,x,y,z)=0 and max(w,x,y,z)=k
yrnsmurf
2016-02-26 20:58:22
At least 1 must be $p^k$ and 1 must be $p^0$
At least 1 must be $p^k$ and 1 must be $p^0$
copeland
2016-02-26 20:58:27
We need to have $0 \le w,x,y,z \le k$, with at least one of them 0 (to force gcd 1) and at least one of them $k$ (to force the lcm to have the prime factor $p^k$).
We need to have $0 \le w,x,y,z \le k$, with at least one of them 0 (to force gcd 1) and at least one of them $k$ (to force the lcm to have the prime factor $p^k$).
copeland
2016-02-26 20:58:39
How do we count this?
How do we count this?
copeland
2016-02-26 20:59:40
There's a keyword "at least" here.
There's a keyword "at least" here.
copeland
2016-02-26 20:59:42
What's that tell us?
What's that tell us?
agbdmrbirdyface
2016-02-26 21:00:09
complementary counting?
complementary counting?
W.Sun
2016-02-26 21:00:09
Complimentary counting
Complimentary counting
Tapeman
2016-02-26 21:00:09
complementary
complementary
mathguy5041
2016-02-26 21:00:09
Complementary counting?
Complementary counting?
weeshree
2016-02-26 21:00:09
Complimentary
Complimentary
LetThePlatPlay
2016-02-26 21:00:09
complementary counting
complementary counting
copeland
2016-02-26 21:00:15
Often that's the cue for complementary counting.
Often that's the cue for complementary counting.
copeland
2016-02-26 21:00:19
It seems hard here.
It seems hard here.
copeland
2016-02-26 21:00:26
Is there another technique that we can use?
Is there another technique that we can use?
MSTang
2016-02-26 21:00:40
PIE
PIE
temp8909
2016-02-26 21:00:40
PIE and complmentary counting!!!
PIE and complmentary counting!!!
eveningstarandlion
2016-02-26 21:00:40
PIE?
PIE?
copeland
2016-02-26 21:00:42
We can use PIE! (The "at least" is often our clue to use PIE.)
We can use PIE! (The "at least" is often our clue to use PIE.)
copeland
2016-02-26 21:01:21
This is a little more subtle than just these names imply. We're kind of using complementary counting and then fixing stuff here, but what is a name, really?
This is a little more subtle than just these names imply. We're kind of using complementary counting and then fixing stuff here, but what is a name, really?
W.Sun
2016-02-26 21:01:29
Principle of inclusion exclusion!
Principle of inclusion exclusion!
copeland
2016-02-26 21:01:49
PIE is the Principle of Inclusion-Exclusion. Check this out, it's cool.
PIE is the Principle of Inclusion-Exclusion. Check this out, it's cool.
copeland
2016-02-26 21:02:00
How many ways to have $(w,x,y,z)$ with $0 \le w,x,y,z \le k$, forgetting about the other conditions?
How many ways to have $(w,x,y,z)$ with $0 \le w,x,y,z \le k$, forgetting about the other conditions?
USA
2016-02-26 21:02:41
What is PIE?
What is PIE?
swagger21
2016-02-26 21:02:42
(k+1)^4
(k+1)^4
mathguy5041
2016-02-26 21:02:42
(k+1)^4?
(k+1)^4?
illogical_21
2016-02-26 21:02:42
(k+1)^4
(k+1)^4
legozelda
2016-02-26 21:02:42
(k+1)^4
(k+1)^4
yrnsmurf
2016-02-26 21:02:42
$(k+1)^4$
$(k+1)^4$
SimonSun
2016-02-26 21:02:42
$(k+1)^4$
$(k+1)^4$
randomsolver
2016-02-26 21:02:42
(k+1)4
(k+1)4
copeland
2016-02-26 21:02:45
Each exponent has $k+1$ choices, so there are $(k+1)^4$ ways.
Each exponent has $k+1$ choices, so there are $(k+1)^4$ ways.
copeland
2016-02-26 21:02:52
Now we have to subtract those with no 0's, and subtract those with no $k$'s.
Now we have to subtract those with no 0's, and subtract those with no $k$'s.
copeland
2016-02-26 21:02:53
How many with no 0's?
How many with no 0's?
mathguy5041
2016-02-26 21:03:22
k^4
k^4
agbdmrbirdyface
2016-02-26 21:03:22
K^4?
K^4?
temp8909
2016-02-26 21:03:22
-k^4
-k^4
pinkrock
2016-02-26 21:03:22
(k)^4
(k)^4
summitwei
2016-02-26 21:03:22
$k^4$
$k^4$
illogical_21
2016-02-26 21:03:22
k^4
k^4
thequantumguy
2016-02-26 21:03:22
k^4
k^4
copeland
2016-02-26 21:03:25
If no 0's, then we have $1 \le w,x,y,z \le k$. Each has $k$ choices, so that's $k^4$ possibilities.
If no 0's, then we have $1 \le w,x,y,z \le k$. Each has $k$ choices, so that's $k^4$ possibilities.
copeland
2016-02-26 21:03:27
If no $k$'s, then we have $0 \le w,x,y,z \le k-1$. Again each has $k$ choices, so that's $k^4$ possibilities.
If no $k$'s, then we have $0 \le w,x,y,z \le k-1$. Again each has $k$ choices, so that's $k^4$ possibilities.
copeland
2016-02-26 21:03:28
So that gives us $(k+1)^4 - 2k^4$ as our count so far. Are we done?
So that gives us $(k+1)^4 - 2k^4$ as our count so far. Are we done?
ein
2016-02-26 21:04:06
double counted those with none of both
double counted those with none of both
randomsolver
2016-02-26 21:04:06
(k-1)^4
(k-1)^4
sillyd
2016-02-26 21:04:06
nope add the number without k's or 0's
nope add the number without k's or 0's
summitwei
2016-02-26 21:04:06
No, you still need to add the ones which don't have either of them
No, you still need to add the ones which don't have either of them
copeland
2016-02-26 21:04:08
No -- we've doubly subtracted the $(w,x,y,z)$ with both no 0's and no $k$'s. We only want to subtract them out once, so we need to add them back in.
No -- we've doubly subtracted the $(w,x,y,z)$ with both no 0's and no $k$'s. We only want to subtract them out once, so we need to add them back in.
copeland
2016-02-26 21:04:13
These have $1 \le w,x,y,z \le k-1$, so there are $(k-1)^4$ of them.
These have $1 \le w,x,y,z \le k-1$, so there are $(k-1)^4$ of them.
copeland
2016-02-26 21:04:14
Thus our final count is $(k+1)^4 - 2k^4 + (k-1)^4$.
Thus our final count is $(k+1)^4 - 2k^4 + (k-1)^4$.
copeland
2016-02-26 21:04:15
How does this simplify?
How does this simplify?
swagger21
2016-02-26 21:05:18
12k^2 + 2
12k^2 + 2
illogical_21
2016-02-26 21:05:18
12k^2+2
12k^2+2
agbdmrbirdyface
2016-02-26 21:05:18
oh wait it just comes down to 12k^2 + 2
oh wait it just comes down to 12k^2 + 2
Jayjayliu
2016-02-26 21:05:18
12*k^2+2
12*k^2+2
thequantumguy
2016-02-26 21:05:18
12k^2 + 2
12k^2 + 2
copeland
2016-02-26 21:05:23
Rather nicely! Most of the terms cancel, and we're left with just $12k^2 + 2$.
Rather nicely! Most of the terms cancel, and we're left with just $12k^2 + 2$.
copeland
2016-02-26 21:05:24
(Incidentally, this is the "second discrete derivative of $k^4$", so we know that $4\cdot3 k^2$ is the right leading term.)
(Incidentally, this is the "second discrete derivative of $k^4$", so we know that $4\cdot3 k^2$ is the right leading term.)
copeland
2016-02-26 21:05:26
So, to recap, if we have $(a,b,c,d)$ with gcd 1 and a factor of $p^k$ in the lcm, then there are $12k^2 + 2$ ways to allocate powers of $p$ to the four numbers.
So, to recap, if we have $(a,b,c,d)$ with gcd 1 and a factor of $p^k$ in the lcm, then there are $12k^2 + 2$ ways to allocate powers of $p$ to the four numbers.
copeland
2016-02-26 21:05:28
How do we use this to get 77,000?
How do we use this to get 77,000?
SherlockHolmes7
2016-02-26 21:06:26
set it equal to 77000
set it equal to 77000
agbdmrbirdyface
2016-02-26 21:06:26
set it equal to 77000?
set it equal to 77000?
copeland
2016-02-26 21:06:39
Almost, except what are we forgetting?
Almost, except what are we forgetting?
yrnsmurf
2016-02-26 21:07:16
$(12k^2+2)(12l^2+2)(12m^2+2)$etc.=77000
$(12k^2+2)(12l^2+2)(12m^2+2)$etc.=77000
swagger21
2016-02-26 21:07:16
theres more than one prime factor most likely
theres more than one prime factor most likely
copeland
2016-02-26 21:07:20
Ah.
Ah.
copeland
2016-02-26 21:07:22
We do this separately for each prime power in the lcm, and then multiply them together.
We do this separately for each prime power in the lcm, and then multiply them together.
copeland
2016-02-26 21:07:24
So we're looking for values of $k_1,k_2,\ldots$ (possibly repeated) that we can multiply together the $(12k_i^2 + 2)\text{'s}$ to get 77,000.
So we're looking for values of $k_1,k_2,\ldots$ (possibly repeated) that we can multiply together the $(12k_i^2 + 2)\text{'s}$ to get 77,000.
copeland
2016-02-26 21:07:26
It might help to compute some values of $12k^2 + 2$, and compare to the factors of 77,000.
It might help to compute some values of $12k^2 + 2$, and compare to the factors of 77,000.
copeland
2016-02-26 21:07:31
First, note that $77000 = 77 \cdot 1000 = 2^3 \cdot 5^3 \cdot 7 \cdot 11$.
First, note that $77000 = 77 \cdot 1000 = 2^3 \cdot 5^3 \cdot 7 \cdot 11$.
copeland
2016-02-26 21:07:33
Then, we have, for $1 \le k \le 5$:
\[\begin{array}{r||c|c|c|c|c}
k & 1 & 2 & 3 & 4 & 5 & \\ \hline
12k^2 + 2 & 14 & 50 & 110 & 194 & 302
\end{array}\]
Then, we have, for $1 \le k \le 5$:
\[\begin{array}{r||c|c|c|c|c}
k & 1 & 2 & 3 & 4 & 5 & \\ \hline
12k^2 + 2 & 14 & 50 & 110 & 194 & 302
\end{array}\]
copeland
2016-02-26 21:07:36
Can we see any combination of these that multiply to give 77000?
Can we see any combination of these that multiply to give 77000?
summitwei
2016-02-26 21:08:34
You need one power of 1, one of 2, and one of 3, because $(12*1^2+2)(12*2^2+2)(12*3^2+2)=77000
You need one power of 1, one of 2, and one of 3, because $(12*1^2+2)(12*2^2+2)(12*3^2+2)=77000
mathwizard888
2016-02-26 21:08:34
14*50*110
14*50*110
summitwei
2016-02-26 21:08:34
14*50*110=77000
14*50*110=77000
ninjataco
2016-02-26 21:08:34
14*50*110
14*50*110
agbdmrbirdyface
2016-02-26 21:08:34
110, 14, and 50
110, 14, and 50
ein
2016-02-26 21:08:34
14, 50. 110
14, 50. 110
sillyd
2016-02-26 21:08:34
14*50*110=77,000
14*50*110=77,000
illogical_21
2016-02-26 21:08:34
110,14,50
110,14,50
LetThePlatPlay
2016-02-26 21:08:34
14*50*110?
14*50*110?
yrnsmurf
2016-02-26 21:08:34
110 and 50 and 14
110 and 50 and 14
illogical_21
2016-02-26 21:08:34
110*14*50=77000
110*14*50=77000
Jayjayliu
2016-02-26 21:08:34
1,2,3
1,2,3
SK200
2016-02-26 21:08:34
14*50*110
14*50*110
techguy2
2016-02-26 21:08:34
14*50*110
14*50*110
thequantumguy
2016-02-26 21:08:34
14,15,110
14,15,110
copeland
2016-02-26 21:08:36
$k=1$ is the only one so far with a factor of 7, and $k=3$ is the only one with a factor of 11, so we'll probably need those.
$k=1$ is the only one so far with a factor of 7, and $k=3$ is the only one with a factor of 11, so we'll probably need those.
copeland
2016-02-26 21:08:37
And there's not enough 5's in 110, but we can't take any more 11's, so we'll need a $k=2$ term too.
And there's not enough 5's in 110, but we can't take any more 11's, so we'll need a $k=2$ term too.
copeland
2016-02-26 21:08:38
And that's enough! $14 \cdot 50 \cdot 110 = 77000$.
And that's enough! $14 \cdot 50 \cdot 110 = 77000$.
copeland
2016-02-26 21:08:39
So what have we concluded?
So what have we concluded?
sillyd
2016-02-26 21:10:11
m=2^3 * 3^2 * 5
m=2^3 * 3^2 * 5
yrnsmurf
2016-02-26 21:10:11
$k^1l^2m^3$
$k^1l^2m^3$
thequantumguy
2016-02-26 21:10:11
theres a 3,2,1 in the exponents so u put the larger exponents on the smaller bases 2, 3, 5
theres a 3,2,1 in the exponents so u put the larger exponents on the smaller bases 2, 3, 5
copeland
2016-02-26 21:10:14
We use one prime with $k=1$, one prime with $k=2$, and one prime with $k=3$.
We use one prime with $k=1$, one prime with $k=2$, and one prime with $k=3$.
copeland
2016-02-26 21:10:17
That is, if $m$ is of the form $pq^2r^3$, then the number of quadruples $(a,b,c,d)$ with the desired property is exactly 77,000.
That is, if $m$ is of the form $pq^2r^3$, then the number of quadruples $(a,b,c,d)$ with the desired property is exactly 77,000.
copeland
2016-02-26 21:10:18
What's the smallest this can be?
What's the smallest this can be?
agbdmrbirdyface
2016-02-26 21:11:11
so then it has to be 2^3 3^2 * 5
so then it has to be 2^3 3^2 * 5
Superwiz
2016-02-26 21:11:11
D
D
LetThePlatPlay
2016-02-26 21:11:11
D
D
swagger21
2016-02-26 21:11:11
360
360
weeshree
2016-02-26 21:11:11
360
360
copeland
2016-02-26 21:11:14
We want the higher powers to be as small a prime as possible.
We want the higher powers to be as small a prime as possible.
copeland
2016-02-26 21:11:15
So we want $m = 2^3 \cdot 3^2 \cdot 5 = 8 \cdot 9 \cdot 5 = 360$.
So we want $m = 2^3 \cdot 3^2 \cdot 5 = 8 \cdot 9 \cdot 5 = 360$.
copeland
2016-02-26 21:11:23
In the original problem, we get $n = 360 \cdot 77 = \boxed{27720}$. Answer (D).
In the original problem, we get $n = 360 \cdot 77 = \boxed{27720}$. Answer (D).
copeland
2016-02-26 21:11:31
Alright, it looks like we have, um, 54 points.
Alright, it looks like we have, um, 54 points.
copeland
2016-02-26 21:11:39
Anybody want to give up now?
Anybody want to give up now?
weeshree
2016-02-26 21:11:49
60!
60!
agbdmrbirdyface
2016-02-26 21:11:49
25 lezzgo
25 lezzgo
W.Sun
2016-02-26 21:11:49
NO
NO
agbdmrbirdyface
2016-02-26 21:11:49
no
no
temp8909
2016-02-26 21:11:49
no!
no!
eveningstarandlion
2016-02-26 21:11:49
That's not AIME qualification, so no
That's not AIME qualification, so no
liuh008
2016-02-26 21:11:49
NEVER
NEVER
agbdmrbirdyface
2016-02-26 21:11:49
not in the slightest
not in the slightest
copeland
2016-02-26 21:12:01
Alright, let's rock this so-called number 25.
Alright, let's rock this so-called number 25.
copeland
2016-02-26 21:12:04
25. The sequence $(a_n)$ is defined recursively by $a_0 = 1$, $a_1 = \sqrt[19]{2}$, and $a_n = a_{n-1}a_{n-2}^2$ for $n \ge 2$. What is the smallest positive integer $k$ such that the product $a_1a_2\cdots a_k$ is an integer?
$\phantom{12B:25}$
(A) 17 (B) 18 (C) 19 (D) 20 (E) 21
25. The sequence $(a_n)$ is defined recursively by $a_0 = 1$, $a_1 = \sqrt[19]{2}$, and $a_n = a_{n-1}a_{n-2}^2$ for $n \ge 2$. What is the smallest positive integer $k$ such that the product $a_1a_2\cdots a_k$ is an integer?
$\phantom{12B:25}$
(A) 17 (B) 18 (C) 19 (D) 20 (E) 21
copeland
2016-02-26 21:12:06
19th roots, ick. What makes them go away?
19th roots, ick. What makes them go away?
copeland
2016-02-26 21:13:18
Exponentiation does make them go away.
Exponentiation does make them go away.
copeland
2016-02-26 21:13:19
What else?
What else?
copeland
2016-02-26 21:13:31
We can't take the 19th power since we always get integers that way.
We can't take the 19th power since we always get integers that way.
mathwizard888
2016-02-26 21:13:40
take log2, becomes linear recurrence
take log2, becomes linear recurrence
LetThePlatPlay
2016-02-26 21:13:40
just express each term as 2^(1/19)^n
just express each term as 2^(1/19)^n
temp8909
2016-02-26 21:13:43
logarithms!
logarithms!
yrnsmurf
2016-02-26 21:13:43
Logarithms
Logarithms
copeland
2016-02-26 21:13:46
Let's take the logarithm base 2 of everything.
Let's take the logarithm base 2 of everything.
copeland
2016-02-26 21:13:47
That is, let's define $b_n = \log_2 a_n$.
That is, let's define $b_n = \log_2 a_n$.
copeland
2016-02-26 21:13:50
How does this rephrase the problem?
How does this rephrase the problem?
agbdmrbirdyface
2016-02-26 21:14:54
a0 = 0, a1 = 1/19
a0 = 0, a1 = 1/19
temp8909
2016-02-26 21:14:54
$b_0=1,b_1=\frac{1}{19},\text{ and }b_n=b_{n-1}+2b_{n-2}$
$b_0=1,b_1=\frac{1}{19},\text{ and }b_n=b_{n-1}+2b_{n-2}$
ninjataco
2016-02-26 21:14:54
b_1 + b_2 + ... + b_k is multiple of 19
b_1 + b_2 + ... + b_k is multiple of 19
agbdmrbirdyface
2016-02-26 21:14:54
and bn = bn-1 + 2bn-2
and bn = bn-1 + 2bn-2
weeshree
2016-02-26 21:14:57
an = a_n-1 + 2a_n-2
an = a_n-1 + 2a_n-2
copeland
2016-02-26 21:14:59
Now multiplication is addition!
Now multiplication is addition!
copeland
2016-02-26 21:15:05
And our problem looks more like:
And our problem looks more like:
copeland
2016-02-26 21:15:06
25X. The sequence $(b_n)$ is defined recursively by $b_0 = 0$, $b_1 = \dfrac{1}{19}$, and $b_n = b_{n-1} + 2b_{n-2}$ for $n \ge 2$. What is the smallest positive integer $k$ such that the sum $b_1 + b_2 + \cdots + b_k$ is an integer?
$\phantom{12B:25X}$
(A) 17 (B) 18 (C) 19 (D) 20 (E) 21
25X. The sequence $(b_n)$ is defined recursively by $b_0 = 0$, $b_1 = \dfrac{1}{19}$, and $b_n = b_{n-1} + 2b_{n-2}$ for $n \ge 2$. What is the smallest positive integer $k$ such that the sum $b_1 + b_2 + \cdots + b_k$ is an integer?
$\phantom{12B:25X}$
(A) 17 (B) 18 (C) 19 (D) 20 (E) 21
copeland
2016-02-26 21:15:10
Note that the product of the $a$'s in the original problem is an integer if and only if the sum of the $b$'s in the new problem is an integer.
Note that the product of the $a$'s in the original problem is an integer if and only if the sum of the $b$'s in the new problem is an integer.
copeland
2016-02-26 21:15:12
What else would simplify things?
What else would simplify things?
yrnsmurf
2016-02-26 21:15:48
Multiplying by 19
Multiplying by 19
yrnsmurf
2016-02-26 21:15:48
and also mods
and also mods
TheRealDeal
2016-02-26 21:15:48
multiply by 19
multiply by 19
MSTang
2016-02-26 21:15:48
Set $c_k = 19b_k$
Set $c_k = 19b_k$
summitwei
2016-02-26 21:15:48
Let $b_1=x$ and look for a multiple of 19
Let $b_1=x$ and look for a multiple of 19
copeland
2016-02-26 21:15:50
Everything in sight is a multiple of $\frac{1}{19}$. So we could multiply by 19, and it's necessary and sufficient for the sum to be a multiple of 19.
Everything in sight is a multiple of $\frac{1}{19}$. So we could multiply by 19, and it's necessary and sufficient for the sum to be a multiple of 19.
copeland
2016-02-26 21:15:51
That is, we can change the problem again:
That is, we can change the problem again:
copeland
2016-02-26 21:15:52
25Y. The sequence $(c_n)$ is defined recursively by $c_0 = 0$, $c_1 = 1$, and $c_n = c_{n-1} + 2c_{n-2}$ for $n \ge 2$. What is the smallest positive integer $k$ such that the sum $c_1 + c_2 + \cdots + c_k \equiv 0 \pmod{19}$?
$\phantom{12B:25Y}$
(A) 17 (B) 18 (C) 19 (D) 20 (E) 21
25Y. The sequence $(c_n)$ is defined recursively by $c_0 = 0$, $c_1 = 1$, and $c_n = c_{n-1} + 2c_{n-2}$ for $n \ge 2$. What is the smallest positive integer $k$ such that the sum $c_1 + c_2 + \cdots + c_k \equiv 0 \pmod{19}$?
$\phantom{12B:25Y}$
(A) 17 (B) 18 (C) 19 (D) 20 (E) 21
copeland
2016-02-26 21:15:53
Seems simple enough now.
Seems simple enough now.
copeland
2016-02-26 21:15:59
Now what should we do?
Now what should we do?
LetThePlatPlay
2016-02-26 21:16:43
just start plugging in
just start plugging in
summitwei
2016-02-26 21:16:43
Start listing out some numbers
Start listing out some numbers
weeshree
2016-02-26 21:16:43
bash out!
bash out!
TheRealDeal
2016-02-26 21:16:43
plug in numbers up to 21 XD
plug in numbers up to 21 XD
copeland
2016-02-26 21:16:45
Yeah, wait, those answers are really small. Let's just start computing. . .
Yeah, wait, those answers are really small. Let's just start computing. . .
copeland
2016-02-26 21:16:54
What do you get?
What do you get?
copeland
2016-02-26 21:16:56
Indeed, here's the table mod 19:
\[\begin{array}{r|c|c}
k & c_k & c_1 + \cdots + c_k \\ \hline
1 & 1 & 1 \\ \hline
2 & 1 & 2 \\ \hline
3 & 3 & 5 \\ \hline
4 & 5 & 10 \\ \hline
5 & 11 & 2 \\ \hline
6 & 2 & 4 \\ \hline
7 & 5 & 9 \\ \hline
8 & 9 & 18 \\ \hline
9 & 0 & 18 \\ \hline
10 & 18 & 17 \\ \hline
11 & 18 & 16 \\ \hline
12 & 16 & 13 \\ \hline
13 & 14 & 8 \\ \hline
14 & 8 & 16 \\ \hline
15 & 17 & 14 \\ \hline
16 & 14 & 9 \\ \hline
17 & 10 & 0
\end{array}\]
Indeed, here's the table mod 19:
\[\begin{array}{r|c|c}
k & c_k & c_1 + \cdots + c_k \\ \hline
1 & 1 & 1 \\ \hline
2 & 1 & 2 \\ \hline
3 & 3 & 5 \\ \hline
4 & 5 & 10 \\ \hline
5 & 11 & 2 \\ \hline
6 & 2 & 4 \\ \hline
7 & 5 & 9 \\ \hline
8 & 9 & 18 \\ \hline
9 & 0 & 18 \\ \hline
10 & 18 & 17 \\ \hline
11 & 18 & 16 \\ \hline
12 & 16 & 13 \\ \hline
13 & 14 & 8 \\ \hline
14 & 8 & 16 \\ \hline
15 & 17 & 14 \\ \hline
16 & 14 & 9 \\ \hline
17 & 10 & 0
\end{array}\]
copeland
2016-02-26 21:17:09
So. . .
So. . .
temp8909
2016-02-26 21:17:54
17
17
math101010
2016-02-26 21:17:54
17 yay
17 yay
thequantumguy
2016-02-26 21:17:54
17 WORKS!!!!
17 WORKS!!!!
SimonSun
2016-02-26 21:17:54
17 A
17 A
agbdmrbirdyface
2016-02-26 21:17:54
oh jeez copeland
oh jeez copeland
lucasxia01
2016-02-26 21:17:54
17 !
17 !
SineWarrior
2016-02-26 21:17:54
so 17 is the answer?
so 17 is the answer?
ninjataco
2016-02-26 21:17:54
(A)
(A)
illogical_21
2016-02-26 21:17:54
17
17
techguy2
2016-02-26 21:17:54
17!
17!
baseballcat
2016-02-26 21:17:54
A?
A?
MSTang
2016-02-26 21:17:54
A, gg.
A, gg.
agbdmrbirdyface
2016-02-26 21:17:54
A.... but that seems kinda cheap
A.... but that seems kinda cheap
copeland
2016-02-26 21:17:57
We see that $k = \boxed{17}$ is the first time the right column in 0. Answer (A).
We see that $k = \boxed{17}$ is the first time the right column in 0. Answer (A).
copeland
2016-02-26 21:18:03
Yeah, huh?
Yeah, huh?
copeland
2016-02-26 21:18:18
We came up with a few ways to make this problem harder, but they just made it harder. . .
We came up with a few ways to make this problem harder, but they just made it harder. . .
summitwei
2016-02-26 21:18:26
Also, $c_9=-1$ and $c_10=-2$, so you can simplify some of the calculation
Also, $c_9=-1$ and $c_10=-2$, so you can simplify some of the calculation
copeland
2016-02-26 21:18:43
Yes, in fact, when I solved this problem I used negative numbers liberally.
Yes, in fact, when I solved this problem I used negative numbers liberally.
techguy2
2016-02-26 21:18:47
wow, were done with.... 60 points! wow. not even honor roll.
wow, were done with.... 60 points! wow. not even honor roll.
copeland
2016-02-26 21:18:50
Killer.
Killer.
copeland
2016-02-26 21:18:55
Oh well. . . maybe next year.
Oh well. . . maybe next year.
copeland
2016-02-26 21:19:39
So, that's it for the 2016 AMCs.
So, that's it for the 2016 AMCs.
copeland
2016-02-26 21:19:49
But we'll be back!
But we'll be back!
copeland
2016-02-26 21:19:51
AoPS will hold a Math Jam after each AIME, where we'll discuss all 15 problems from the contest. The schedule is:
AIME I Math Jam: Saturday, March 5, 7 PM Eastern
AIME II Math Jam: Friday, March 18, 7 PM Eastern
We hope to see you there!
AoPS will hold a Math Jam after each AIME, where we'll discuss all 15 problems from the contest. The schedule is:
AIME I Math Jam: Saturday, March 5, 7 PM Eastern
AIME II Math Jam: Friday, March 18, 7 PM Eastern
We hope to see you there!
copeland
2016-02-26 21:20:43
Oh, also: AoPS is running our weekend Special AIME Problem Seminar this upcoming weekend, Feb 27 and 28, from 3:30-6:30 PM Eastern each day. More information is at http://artofproblemsolving.com/school/course/catalog/maa-aime-special.
Oh, also: AoPS is running our weekend Special AIME Problem Seminar this upcoming weekend, Feb 27 and 28, from 3:30-6:30 PM Eastern each day. More information is at http://artofproblemsolving.com/school/course/catalog/maa-aime-special.
copeland
2016-02-26 21:20:52
That is tomorrow.
That is tomorrow.
SK200
2016-02-26 21:20:56
are all math jams at 7pm eastern?
are all math jams at 7pm eastern?
copeland
2016-02-26 21:21:00
No, most are 7:30.
No, most are 7:30.
copeland
2016-02-26 21:21:06
The AMC and AIME ones start early.
The AMC and AIME ones start early.
copeland
2016-02-26 21:21:12
I'll be closing the room shortly. Thanks for coming!
I'll be closing the room shortly. Thanks for coming!
thequantumguy
2016-02-26 21:21:47
goodbye
goodbye
math101010
2016-02-26 21:21:47
Thank you!
Thank you!
SimonSun
2016-02-26 21:21:47
Thks
Thks
jenmath888
2016-02-26 21:21:47
Thanks.
Thanks.
ein
2016-02-26 21:21:47
Thanks!
Thanks!
agbdmrbirdyface
2016-02-26 21:21:47
seeya copeland
seeya copeland
SimonSun
2016-02-26 21:21:47
K bye
K bye
DavidUsa
2016-02-26 21:21:47
THANK YOU
THANK YOU
copeland
2016-02-26 21:21:48
Bye!
Bye!
Copyright © 2024 AoPS Incorporated. This page is copyrighted material. You can view and print this page for your own use, but you cannot share the contents of this file with others.